Цены снижены! Бесплатная доставка контурной маркировки по всей России

Как обозначается угловая скорость в физике: 404 — Страница не найдена

Содержание

Угловое ускорение

Система понятий кинематики включает в себя также такую величину как угловое ускорение тела. Дадим ей определение, рассмотрим основные аспекты с использованием примеров.

Основные понятия

Определение 1

Угловое ускорение – величина, характеризующая изменение скорости с течением времени.

Пусть рассматриваемый промежуток времени это: Δt=t1-t, а изменение угловой скорости составит Δω=ω1-ω, тогда числовое значение среднего углового ускорения за тот же интервал времени: ε=∆ω∆t=ε. Перейдем к пределу, когда Δt>0, тогда формула углового ускорения будет иметь вид: ε=lim∆t→0∆ω∆t=dωdt=d2φdt=ω˙=φ¨.

Определение 2

Числовое значение ускорения в заданный момент времени есть первая производная от угловой скорости или вторая производная от угла поворота по времени.

Размерность углового ускорения 1T2 (т.е. 1время2). Укажем также, в чем измеряется угловое ускорение: за единицу измерения стандартно принимается рад/с2 или иначе: 1с2(с-2).

Определение 3

Ускоренное вращение тела – это вращение, при котором угловая скорость (ее модуль) возрастает с течением времени.

Определение 4

Замедленное вращение тела – это вращение, при котором угловая скорость (ее модуль) убывает с течением времени.

В общем, довольно просто заметить, что, если ω и ε имеют одинаковые знаки, наблюдается ускоренное вращение, а, когда противоположные знаки – замедленное.

Рисунок 1. Вектор углового ускорения

Если мы представим угловое ускорение как вектор ε→=dω→dt, имеющий направление вдоль оси вращения, то в случае ускоренного вращения

ε→ и ω→ совпадут по направлениям (левая часть
рисунка 1) и будут противоположны по направлениям в случае замедленного вращения (правая часть
рисунка 1).

Нужна помощь преподавателя?

Опиши задание — и наши эксперты тебе помогут!

Описать задание

Закон равнопеременного вращения

Определение 5

Равнопеременное вращение – вращение, при котором угловое ускорение во все время движения является постоянным (ε=const).

Выведем формульно закон равнопеременного вращения. Пусть в начальный момент времени t0 угол вращения равен ϕ=ϕ0; угловая скорость — ω=ω0 (т.е. ω0 является начальной угловой скоростью).

Выражение ε=dωdt=ω˙=φ¨ дает нам возможность сделать запись: dω=εdt. Проинтегрируем левую часть крайней записи в пределах от ω0 до ω, а правую – в пределах от 0 до t, тогда:

ω=ω0+εt, dφ=ω0dt+εtdt.

Проинтегрируем вторично и получим формулу, выражающую закон равнопеременного вращения:

Определение 6

Закон равнопеременного вращения: φ=φ0+ωt+εt22.

Вращение является равноускоренным, когда ω и ε имеют одинаковые знаки.

Вращение является равнозамедленным, когда ω и ε противоположны по знаку.

Угловое ускорение имеет связь с полным и тангенциальным ускорениями. Пусть некоторая точка вращается неравномерно по окружности с радиусом R, тогда: αr=εR. Нормальное ускорение имеет также связь с угловым: an=ω2R. Учтем это выражение и для полного ускорения получим: a=ar2+an2=Rε2+ω4 Для равнопеременного движения: ω=εt; an=ω2R=ε2t2R и a=Rε2+ε4t4=Rε1+ε2t4.

Практические примеры

Пример 1

На рисунке 2 заданы различные типы вращения гироскопа (волчка). С учетом соответствующих подписей необходимо указать, какой рисунок верно демонстрирует направление углового ускорения.

Рисунок 2

Решение

Правило буравчика (правого винта) связывает направление вращения и псевдовектор угловой скорости. Рисунки 2.1. и 2.3. показывают направление псевдовектора вверх, а рисунки 2.2. и 2.4. – вниз.

Когда угловая скорость возрастает, ее приращение и вектор ускорения совпадут с вектором угловой скорости (рисунки 2.1. и 2.4.). Когда угловая скорость будет уменьшаться, ее приращение и вектор ускорения окажутся противоположно направлены вектору угловой скорости (рисунки 2.2. и 2.3.). Таким образом, все рисунки демонстрируют верное направление углового ускорения.

Пример 2

Пусть задана некоторая материальная точка, совершающая движение по окружности с радиусом R. При этом выражение ϕ=αt3 отражает зависимость угла поворота от времени.

Необходимо найти полное ускорение заданной точки как функцию времени.

Решение

Запишем выражения для угловой скорости и углового ускорения заданной точки:

ω=dφdt=3αt2; ε=6αt.

Полное ускорение запишем как:

a=ar2+an2=Rε2+ω4=R36a2t2+81a4t8=3atR4+9a2t6.

Период вращения тела по окружности. Равномерное движение по окружности. Угловая скорость и угловое ускорение

>>Физика: Период и частота обращения

Равномерное движение по окружности характеризуют периодом и частотой обращения.

Период обращения — это время, за которое совершается один оборот.

Если, например, за время t = 4 с тело, двигаясь по окружности, совершило n = 2 оборота, то легко сообразить, что один оборот длился 2 с. Это и есть период обращения. Обозначается он буквой Т и определяется по формуле:

Итак, чтобы найти период обращения, надо время, за которое совершено п оборотов, разделить на число оборотов .

Другой характеристикой равномерного движения по окружности является частота обращения.

Частота обращения — это число оборотов, совершаемых за 1 с. Если, например, за время t = 2 с тело совершило n = 10 оборотов, то легко сообразить, что за 1 с оно успевало совершить 5 оборотов. Это число и выражает частоту обращения. Обозначается она греческой буквой

V (читается: ню) и определяется по формуле:

Итак, чтобы найти частоту обращения, надо число оборотов разделить на время, в течение которого они произошли.

За единицу частоты обращения в СИ принимают частоту обращения, при которой за каждую секунду тело совершает один оборот. Эта единица обозначается так: 1/с или с -1 (читается: секунда в минус первой степени). Раньше эту единицу называли «оборот в секунду», но теперь это название считается устаревшим.

Сравнивая формулы (6.1) и (6.2), можно заметить, что период и частота — величины взаимно обратные. Поэтому

Формулы (6. 1) и (6.3) позволяют найти период обращения Т, если известны число n и время оборотов t или частота обращения V . Однако его можно найти и в том случае, когда ни одна из этих величин неизвестна. Вместо них достаточно знать скорость тела V и радиус окружности r, по которой оно движется.

Для вывода новой формулы вспомним, что период обращения — это время, за которое тело совершает один оборот, т. е. проходит путь, равный длине окружности (l окр = 2 П r, где П ≈3,14- число «пи», известное из курса математики). Но мы знаем, что при равномерном движении время находится делением пройденного пути на скорость движения. Таким образом,

Итак, чтобы найти период обращения тела, надо длину окружности, по которой оно движется, разделить на скорость его движения.

??? 1. Что такое период обращения? 2. Как можно найти период обращения, зная время и число оборотов? 3. Что такое частота обращения ? 4. Как обозначается единица частоты? 5. Как можно найти частоту обращения, зная время и число оборотов? 6. Как связаны между собой период и частота обращения? 7. Как можно найти период обращения, зная радиус окружности и скорость движения тела?

Отослано читателями из интернет-сайтов

Сборник конспектов уроков по физике, рефераты на тему из школьной программы. Календарно тематическое планирование. физика 8 класс онлайн, книги и учебники по физике. Школьнику подготовиться к уроку.

Содержание урока конспект урока опорный каркас презентация урока акселеративные методы интерактивные технологии Практика задачи и упражнения самопроверка практикумы, тренинги, кейсы, квесты домашние задания дискуссионные вопросы риторические вопросы от учеников Иллюстрации аудио-, видеоклипы и мультимедиа фотографии, картинки графики, таблицы, схемы юмор, анекдоты, приколы, комиксы притчи, поговорки, кроссворды, цитаты
Дополнения
рефераты статьи фишки для любознательных шпаргалки учебники основные и дополнительные словарь терминов прочие Совершенствование учебников и уроков исправление ошибок в учебнике обновление фрагмента в учебнике элементы новаторства на уроке замена устаревших знаний новыми Только для учителей идеальные уроки календарный план на год методические рекомендации программы обсуждения Интегрированные уроки

Расстояние и время, которое уходит на преодоление этого расстояния, связывает физическое понятие — скорость.

И у человека, как правило, не вызывает вопросов определение этой величины. Все понимают, что двигаться на автомобиле со скоростью 100 км/ч — значит за один час проехать 100 километров.

А как быть, если тело вращается? Например, обычный бытовой вентилятор делает с десяток оборотов в секунду. И в то же время скорость вращения лопастей такова, что их запросто можно остановить рукой без вреда для себя. Земля вокруг своей звезды — Солнца — делает один оборот за целый год, а это более 30 миллионов секунд, но скорость её движения по околозвёздной орбите составляет около 30 километров за одну секунду!

Как же связать привычную скорость с быстротой вращения, как выглядит формула угловой скорости?

Понятие угловой скорости

Понятие угловой скорости используется в изучении законов вращения. Оно применяется ко всем вращающимся телам. Будь то вращение некоторой массы вокруг другой, как в случае с Землёй и Солнцем, или же вращение самого тела вокруг полярной оси (суточное вращение нашей планеты).

Отличие угловой скорости от линейной в том, что она фиксирует изменение угла, а не расстояния в единицу времени. В физике угловую скорость принято обозначать буквой греческого алфавита «омега» — ω.

Классическая формула угловой скорости вращения рассматривается так.

Представим, что вокруг некоторого центра А вращается физическое тело с постоянной скоростью. Его положение в пространстве относительно центра определяется углом φ. В некоторый момент времени t1 рассматриваемое тело находится в точке В. Угол отклонения тела от начального φ1.

Затем тело перемещается в точку С. Оно находится там в момент времени t2. Время, понадобившееся для данного перемещения:

Меняется и положение тела в пространстве. Теперь угол отклонения равен φ2. Изменение угла за период времени ∆t составило:

∆φ = φ2 — φ1.

Теперь формула угловой скорости формулируется следующим образом: угловая скорость определяется как отношение изменения угла ∆φ за время ∆t.

Единицы измерения угловой скорости

Скорость движения тела линейная измеряется в разных величинах. Движение автотранспорта по дорогам привычно указывают в километрах в час, морские суда делают узлы — морские мили в час. Если же рассматривать движение космических тел, то тут чаще всего фигурируют километры в секунду.

Угловая скорость в зависимости от величины и от предмета, который вращается, также измеряется в разных единицах.

Радианы в секунду (рад/с) — классическое мерило скорости в международной системе единиц (СИ). Показывают — на сколько радиан (в одном полном обороте 2 ∙ 3,14 радиан) успевает повернуться тело за одну секунду.

Обороты в минуту (об/мин) — самая распространённая единица для обозначения скоростей вращения в технике. Валы двигателей как электрических, так и автомобильных выдают именно (достаточно посмотреть на тахометр в своём автомобиле) обороты в минуту.

Обороты в секунду (об/с) — используется реже, прежде всего в образовательных целях.

Период обращения

Иногда для определения скорости вращения удобнее пользоваться другим понятием. Периодом обращения принято называть время, за которое некое тело делает оборот 360° (полный круг) вокруг центра вращения. Формула угловой скорости, выраженная через период обращения, принимает вид:

Выражать периодом обращения быстроту вращения тел оправдано в случаях, когда тело вращается относительно медленно. Вернёмся к рассмотрению движения нашей планеты вокруг светила.

Формула угловой скорости позволяет вычислить её, зная период обращения:

ω = 2П/31536000 = 0,000000199238499086111 рад/с.

Глядя на полученный результат, можно понять, почему, рассматривая вращение небесных тел, удобнее пользоваться именно периодом обращения. Человек видит перед собой понятные цифры и наглядно представляет себе их масштаб.

Связь угловой и линейной скоростей

В некоторых задачах должны быть определены линейная и угловая скорость. Формула трансформации проста: линейная скорость тела равняется произведению угловой скорости на радиус вращения. Как это показано на рисунке.

«Работает» выражение и в обратном порядке, с его помощью определяется и угловая скорость. Формула через скорость линейную получается путём несложных арифметических манипуляций.

Обычно, когда говорят о перемещении, мы представляем себе объект, который движется по прямой. Скорость такого движения принято называть линейной, и расчёт ее средней величины выполняется просто: достаточно найти отношение пройденного расстояния к времени, за которое оно было телом преодолено. Если же объект перемещается по окружности, то в этом случае уже определяется не линейная, а Что это за величина и как ее рассчитывают? Об этом как раз и пойдет разговор в данной статье.

Угловая скорость: понятие и формула

Когда движется по окружности, быстроту ее перемещения можно характеризовать величиной угла поворота радиуса, который соединяет движущийся объект с центром данной окружности. Понятно, что эта величина в зависимости от времени постоянно меняется. Быстрота, с которой этот процесс происходит, и есть не что иное, как угловая скорость. Другими словами, это отношение величины отклонения радиус-вектора объекта к промежутку времени, которое потребовалось объекту на совершение такого поворота. Формула угловой скорости (1) может быть записана в таком виде:

w = φ / t, где:

φ — угол поворота радиуса,

t — период времени вращения.

Единицы измерения величины

В международной системе общепринятых единиц (СИ) для характеристики поворотов принято использовать радианы. Поэтому 1 рад/с — основная единица, которая используется в расчетах угловой скорости. В то же время никто не запрещает применять градусы (напомним, что один радиан равен 180/пи, или 57˚18’). Также угловая скорость может выражаться в числе оборотов за минуту или за секунду. Если перемещение по окружности происходит равномерно, то данная величина может быть найдена по формуле (2):

где n — частота вращения.

В противном случае подобно тому, как это делают для обычной скорости, рассчитывают среднюю, или мгновенную угловую скорость. Следует отметить, что рассматриваемая величина является векторной. Для определения ее направления обычно используют которое часто применяется в физике. Вектор угловой скорости направлен в ту же сторону, в которую происходит винта с правой резьбой. Другими словами, он устремлен вдоль оси, вокруг которой вращается тело, в ту сторону, откуда вращение видно происходящим против движения часовой стрелки.

Примеры расчета

Предположим, требуется определить, чему равна линейная и угловая скорость колеса, если известно, что его диаметр равен одному метру, а угол вращения изменяется в соответствии с законом φ=7t. Воспользуемся нашей первой формулой:

w = φ / t = 7t / t = 7 с -1 .

Это и будет искомая угловая скорость. Теперь перейдем к поиску привычной нам быстроты перемещения. Как известно, v = s / t. Учитывая, что s в нашем случае — это колеса (l =2π*r), а 2π — один полный оборот, получается следующее:

v = 2π*r / t = w * r = 7 * 0.5 = 3.5 м/с

Вот еще одна задачка на эту тему. Известно, что на экваторе равен 6370 километров. Требуется определить линейную и угловую быстроту движения точек, находящихся на этой параллели, которое возникает в результате вращения нашей планеты вокруг своей оси. В данном случае нам понадобится вторая формула:

w = 2π*n = 2*3,14 *(1/(24*3600)) = 7,268 *10 -5 рад/с.

Осталось выяснить, чему равна линейная скорость: v = w*r = 7,268 *10 -5 *6370 * 1000 = 463 м/с.

Движение тела по окружности с постоянной по модулю скоростью — это движение, при котором тело за любые равные промежутки времени описывает одинаковые дуги.

Положение тела на окружности определяется радиусом-вектором \(~\vec r\), проведенным из центра окружности. Модуль радиуса-вектора равен радиусу окружности R (рис. 1).

За время Δt тело, двигаясь из точки А в точку В , совершает перемещение \(~\Delta \vec r\), равное хорде АВ , и проходит путь, равный длине дуги l .

Радиус-вектор поворачивается на угол Δφ . Угол выражают в радианах.

Скорость \(~\vec \upsilon\) движения тела по траектории (окружности) направлена по касательной к траектории. Она называется линейной скоростью . Модуль линейной скорости равен отношению длины дуги окружности l к промежутку времени Δt за который эта дуга пройдена:

\(~\upsilon = \frac{l}{\Delta t}.\)

Скалярная физическая величина, численно равная отношению угла поворота радиуса-вектора к промежутку времени, за который этот поворот произошел, называется угловой скоростью :

\(~\omega = \frac{\Delta \varphi}{\Delta t}.\)

В СИ единицей угловой скорости является радиан в секунду (рад/с).

При равномерном движении по окружности угловая скорость и модуль линейной скорости — величины постоянные: ω = const; υ = const.

Положение тела можно определить, если известен модуль радиуса-вектора \(~\vec r\) и угол φ , который он составляет с осью Ox (угловая координата). Если в начальный момент времени t 0 = 0 угловая координата равна φ 0 , а в момент времени t она равна φ , то угол поворота Δφ радиуса-вектора за время \(~\Delta t = t — t_0 = t\) равен \(~\Delta \varphi = \varphi — \varphi_0\). Тогда из последней формулы можно получить кинематическое уравнение движения материальной точки по окружности :

\(~\varphi = \varphi_0 + \omega t.\)

Оно позволяет определить положение тела в любой момент времени t . Учитывая, что \(~\Delta \varphi = \frac{l}{R}\), получаем\[~\omega = \frac{l}{R \Delta t} = \frac{\upsilon}{R} \Rightarrow\]

\(~\upsilon = \omega R\) — формула связи между линейной и угловой скоростью.

Промежуток времени Τ , в течение которого тело совершает один полный оборот, называется периодом вращения :

\(~T = \frac{\Delta t}{N},\)

где N — число оборотов, совершенных телом за время Δt .

За время Δt = Τ тело проходит путь \(~l = 2 \pi R\). Следовательно,

\(~\upsilon = \frac{2 \pi R}{T}; \ \omega = \frac{2 \pi}{T} .\)

Величина ν , обратная периоду, показывающая, сколько оборотов совершает тело за единицу времени, называется частотой вращения :

\(~\nu = \frac{1}{T} = \frac{N}{\Delta t}. \)

Следовательно,

\(~\upsilon = 2 \pi \nu R; \ \omega = 2 \pi \nu .\)

Литература

Аксенович Л. А. Физика в средней школе: Теория. Задания. Тесты: Учеб. пособие для учреждений, обеспечивающих получение общ. сред, образования / Л. А. Аксенович, Н.Н.Ракина, К. С. Фарино; Под ред. К. С. Фарино. — Мн.: Адукацыя i выхаванне, 2004. — C. 18-19.

Движение по окружности – частный случай криволинейного движения. Скорость тела в любой точке криволинейной траектории направлена по касательной к ней (рис.2.1). Скорость как вектор при этом может изменяться и по модулю (величине) и по направлению. Если модуль скоростиостается неизменным, то говорят оравномерном криволинейном движении.

Пусть тело движется по окружности с постоянной по величине скоростью из точки 1 в точку 2.

При этом тело пройдет путь, равный длине дуги ℓ 12 между точками 1 и 2 за времяt. За это же времяtрадиус- векторR, проведенный из центра окружности 0 к точке, повернется на угол Δφ.

Вектор скорости в точке 2 отличается от вектора скорости в точке 1 по направлению на величину ΔV:

;

Для характеристики изменения вектора скорости на величину δv введем ускорение:

(2.4)

Вектор в любой точке траектории направлен по радиусуRкцентру окружности перпендикулярно к вектору скоростиV 2 . Поэтому ускорение, характеризующее при криволинейном движении изменение скоростипо направлению, называютцентростремительным или нормальным . Таким образом, движение точки по окружности с постоянной по модулю скоростью являетсяускоренным .

Если скорость изменяется не только по направлению, но и по модулю (величине), то кроме нормального ускорениявводят еще икасательное (тангенциальное) ускорение, которое характеризует изменение скорости по величине:

или

Направлен вектор по касательной в любой точке траектории (т.е. совпадает с направлением вектора). Угол между векторамииравен 90 0 .

Полное ускорение точки, движущейся по криволинейной траектории, определяется как векторная сумма (рис. 2.1.).

.

Модуль вектора
.

Угловая скорость и угловое ускорение

При движении материальной точки по окружности радиус-векторR, проведенный из центра окружности О к точке, поворачивается на угол Δφ (рис.2.1). Для характеристики вращения вводятся понятия угловой скорости ω и углового ускорения ε.

Угол φ можно измерять в радианах. 1 рад равен углу, который опирается на дугу ℓ, равную радиусуRокружности, т.е.

или 12 = R φ (2.5.)

Продифференцируем уравнение (2.5.)

(2.6.)

Величина dℓ/dt=V мгн. Величину ω =dφ/dtназываютугловой скоростью (измеряется в рад/с). Получим связь между линейной и угловой скоростями:

Величина ω векторная. Направление вектораопределяетсяправилом винта (буравчика) : оно совпадает с направлением перемещения винта, ориентированного вдоль оси вращения точки или тела и вращаемого в направлении поворота тела (рис.2.2), т.е.
.

Угловым ускорением называется векторная величина производная от угловой скорости (мгновенное угловое ускорение)

, (2.8.)

Вектор совпадает с осью вращения и направлен в туже сторону, что и вектор, если вращение ускоренное, и в противоположную, если вращение замедленное.

Число оборотов n тела в единицу времени называют частотой вращения .

Время Т одного полного оборота тела называют периодом вращения . При этом R опишет угол Δφ=2π радиан

С учетом сказанного

, (2.9)

Уравнение (2.8) можно записать следующим образом:

(2.10)

Тогда тангенциальная составляющая ускорения

а  =R(2.11)

Нормальное ускорение а n можно выразить следующим образом:

с учетом (2.7) и (2.9)

(2.12)

Тогда полное ускорение .

Для вращательного движения с постоянным угловым ускорением можно записать уравнение кинематики по аналогии с уравнением (2.1) – (2.3) для поступательного движения:

,

.

Глава 7. Движемся по орбитам – FIZI4KA

В этой главе. . .

  • Постигаем равномерное вращательное движение
  • Изучаем угловое ускорение
  • Испытываем влияние центростремительной силы
  • Учитываем перемещение, скорость и ускорение
  • Движемся по орбите под действием законов Ньютона и силы гравитационного притяжения
  • Поддерживаем вращение в вертикальной плоскости

Вращательное движение выполняют искусственные спутники вокруг планет, гоночные автомобили по трекам и даже пчелы вокруг ульев. В предыдущих разделах рассматривались такие характеристики прямолинейного движения, как перемещение, скорость и ускорение. В этой главе мы снова рассмотрим их, но теперь уже для вращательного движения.

Для перечисленных выше характеристик прямолинейного движения есть аналоги, характеризующие вращательное движение, а именно: угловое перемещение, угловая скорость и угловое ускорение. Как видно из их названия, роль перемещения во вращательном движении играет угол. Угловая скорость обозначает величину угла поворота за единицу времени, а угловое ускорение — изменение угловой скорости за единицу времени. Все, что нужно сделать, чтобы освоить премудрости вращательного движения, это взять уравнения прямолинейного движения и заменить в них одни характеристики другими: перемещение поменять на угол, скорость — на угловую скорость и ускорение — на угловое ускорение.

Держим курс: равномерное вращательное движение

Если объект движется с постоянной по величине скоростью по окружности, то такое движение называется равномерным вращательным движением. Примерами такого движения являются движение гоночного автомобиля по круглому треку и стрелки на циферблате часов. На рис. 7.1 показан мяч для игры в гольф, привязанный нитью к шесту и совершающий движение по окружности. Мяч совершает движение с одинаковой по величине скоростью, но с изменяющимся направлением. Потому такое движение мяча называется равномерным вращательным движением.

Время, которое требуется мячику (или какому-либо другому объекту), чтобы полностью обогнуть окружность, называется периодом и обозначается символом ​\( T \)​. Период и линейную скорость можно легко связать, если известно пройденное расстояние, т.е. длина окружности ​\( 2\pi r \)​, а точнее ее радиус ​\( r \)​. Итак, линейная скорость мячика ​\( v \)​ равна:

а период вращения ​\( T \)​ равен:

Допустим, что длина нити равна 1 м, а период вращения равен 0,5 с. Чему в таком случае будет равна линейная скорость мячика? Подставим численные значения в одно из предыдущих соотношений и получим:

Итак, мячик вращается с линейной скоростью 13 м/с!

Меняем направление: центростремительное ускорение

При вращательном движении по окружности линейная скорость мячика постоянно меняет направление, как показано на рис. 7.2. Ускорение, характеризующее такое изменение скорости, называется центростремительным (или центробежным). В любой точке вращательного движения с постоянной величиной и меняющимся направлением вектор линейной скорости перпендикулярен радиусу.

Это правило справедливо для всех объектов: вектор линейной скорости объекта, равномерно вращающегося по окружности, всегда перпендикулярен радиусу окружности.

Если в показанных на рис. 7.2 положениях нить, удерживающая мяч, оборвется, то куда полетит мяч? Если в этот момент вектор линейной скорости направлен влево, то мяч полетит влево, а если этот вектор направлен вправо, то мяч полетит вправо, и т.д. Этот, казалось бы, простой и интуитивно понятный момент часто вызывает трудности у тех, кто впервые постигает физику.

Всегда следует помнить, что вектор линейной скорости объекта, выполняющего равномерное вращательное движение, всегда направлен под прямым углом к радиусу вращения в текущей точке траектории. (В общем случае неравномерного криволинейного движения эта компонента вектора скорости, перпендикулярная радиусу вращения и касательная к траектории движения, называется тангенциальной компонентой, а перпендикулярная ей компонента — нормальной компонентой. — Примеч. ред.)

Управляем скоростью с помощью центростремительного ускорения

Особенностью равномерного вращательного движения является постоянство величины линейной скорости. Это значит, что вектор ускорения не имеет компоненты, параллельной вектору линейной скорости, поскольку в противном случае величина линейной скорости менялась бы. Однако при равномерном вращательном движении меняется только направление линейной скорости. Такое изменение линейной скорости поддерживается центростремительным ускорением, направленным к центру окружности вращения и перпендикулярно вектору линейной скорости.

В примерах на рис. 7.1 и 7.2 на мяч со стороны нити действует сила натяжения нити, которая поддерживает его движение по окружности. Именно эта сила сообщает мячу центростремительное ускорение ​\( a_ц \)​, вектор которого показан на рис. 7.1. (Попробуйте раскрутить мяч с помощью привязанной к нему нити, и вы сразу же почувствуете действие этой силы со стороны нити.)

Часто возникает вопрос: если вектор ускорения мяча направлен к центру окружности, то почему мяч не движется к центру? Дело в том, что при равномерном вращательном движении это ускорение меняет только направление, а не величину линейной скорости.

Определяем величину центростремительного ускорения

Нам уже известно направление вектора центростремительного ускорения, а чему же равна его величина? Итак, величина центростремительного ускорения объекта, равномерно движущегося с линейной скоростью ​\( v \)​ по окружности с радиусом ​\( r \)​, равна:

Как видите, величина центростремительного ускорения обратно пропорциональна радиусу окружности ​\( r \)​ и прямо пропорциональна квадрату скорости ​\( v \)​. Поэтому не удивительно, что автомобиль на более крутых поворотах испытывает более сильное центростремительное ускорение.

Стремимся к центру: центростремительная сила

На крутых поворотах действие центростремительного ускорения обеспечивается трением шин по дороге. Какую силу нужно приложить, чтобы удержать движущийся со скоростью ​\( v \)​ автомобиль на повороте с радиусом кривизны ​\( r \)​?

Допустим, что в примере на рис. 7.1 легкий мяч заменили на тяжелое пушечное ядро. Теперь, чтобы поддерживать движение ядра по окружности с тем же радиусом и периодом вращения, потребуется гораздо большая сила.

Дело в том, что сила ​\( F=ma \)​ равна произведению ускорения ​\( a \)​ и массы ​\( m \)​, а значит, увеличение массы объекта (замена мяча на ядро) неизбежно приводит к необходимости увеличения силы для обеспечения прежнего ускорения.

Центростремительная сила ​\( F_ц \)​, необходимая для равномерного вращения по окружности с радиусом ​\( r \)​ объекта массой ​\( m \)​ с постоянной скоростью ​\( v \)​, равна:

С помощью этого уравнения можно легко определить силу, необходимую для равномерного вращения объекта по окружности с известной массой, скоростью и радиусом окружности.

Обратите внимание, что если объект движется по той же окружности, но с разной скоростью, то он будет испытывать разную центростремительную силу.

В примерах на рис. 7.1 и 7.2 мяч движется со скоростью ​\( v \)​ = 13 м/с и удерживается нитью длиной 1,0 м, т.е. в данном случае радиус окружности ​\( r \)​ = 1 м. Какая сила потребуется, чтобы поддерживать такое же движение для пушечного ядра с массой 10 кг? Подставляя численные значения в уже известную нам формулу, получим:

Приличная сила! Остается только надеяться, что ваши руки достаточно сильны, чтобы удержать ядро.

Является ли центростремительная сила реальной силой?

Центростремительная сила не является каким-то особым типом взаимодействия. Она имеет отношение только к объекту, движущемуся по криволинейной траектории, и необходима для удержания объекта на данной траектории. Поэтому ее часто называют центростремительно-необходимой силой. Довольно часто новички считают центростремительную силу каким-то новым фундаментальным типом взаимодействия. И это понятно, поскольку известные нам силы (например, сила гравитации и сила трения) имеют вполне определенный источник, который не зависит от траектории движения. Но это совсем не так для центростремительной силы. Центростремительная сила возникает из необходимости удержания объекта на криволинейной траектории. Сумма всех остальных сил, действующих на объект, который движется по криволинейной траектории, должна быть равна центростремительной силе. (Если объект движется по прямолинейной траектории, а затем ему нужно изменить направление движения, то для этого придется приложить силу, равную центростремительной силе. — Примеч. ред.)

Вписываемся в повороты: учитываем радиус и наклон

Если вам приходилось ехать на автомобиле или велосипеде или даже бежать трусцой, то наверняка вы заметили, что в крутой поворот проще вписаться, если поверхность дороги немного наклонена внутрь поворота. Из опыта известно, что чем больше наклон, тем проще вписаться в поворот. Это объясняется тем, что в таком случае на вас действует меньшая центростремительная сила. Центростремительная сила обеспечивается силой трения о поверхность дороги. Если поверхность дороги покрыта льдом, то сила трения становится меньше и потому часто не удается вписаться в поворот на обледеневшей дороге на большой скорости.

Представьте, что автомобилю с массой 1000 кг нужно вписаться в поворот с радиусом Юм, а коэффициент трения покоя (подробнее о нем см. главу6) равен 0,8. (Здесь используется коэффициент трения покоя, поскольку предполагается, что шины по поверхности дороги.) Какую максимальную скорость может развить этот автомобиль без риска не вписаться в поворот. Итак, сила трения покоя шин о поверхность дороги ​\( F_{трение\,покоя} \)​ должна обеспечивать центростремительную силу:

где ​\( m \)​ — это масса автомобиля, ​\( v \)​ — его скорость, ​\( r \)​ — радиус, ​\( \mu_п \)​ — коэффициент трения покоя, a ​\( g \)​ = 9,8 м/с2 — ускорение свободного падения под действием силы гравитации. Отсюда легко находим скорость:

(Обратите внимание, что максимальная безопасная скорость прохождения поворота не зависит от массы автомобиля. — Примеч. ред.)

Это выражение выглядит очень просто, а после подстановки в него численных значений получим:

Итак, максимальная скорость безопасного проезда при таком повороте равна 8,9 м/с. Пересчитаем в единицы “км/ч”, в которых скорость указана на спидометре, и сравним. Получается, что 8,9 м/с = 32 км/ч, а на спидометре всего 29 км/ч. Прекрасно, но далеко не все водители умеют так быстро рассчитывать безопасную скорость прохождения поворотов. Поэтому конструкторы дорог часто строят повороты с наклоном внутрь, чтобы обеспечить центростремительное ускорение не только за счет силы трения, но и за счет горизонтальной компоненты силы гравитации.

На рис. 7.3 показан пример поворота дороги с некоторым наклоном под углом ​\( \theta \)​ к горизонтали. Предположим, что конструкторы решили полностью обеспечить центростремительное ускорение только за счет горизонтальной компоненты силы гравитации (т.е. без учета силы трения) ​\( F_н\sin\theta \)​, где ​\( F_н \)​ — это нормальная сила (подробнее о ней см. в главе 6). Тогда:

В вертикальном направлении на автомобиль действует сила гравитации ​\( mg \)​, которая уравновешивается вертикальной компонентой нормальной силы \( F_н\cos\theta \):

или, иначе выражая это соотношение, получим:

Подставляя это выражение в прежнее соотношение между центростремительной силой и нормальной силой, получим:

Поскольку ​\( \sin\theta/\!\cos\theta=tg\,\theta \)​ в то

Отсюда легко получаем, что угол наклона поворота дороги ​\( \theta \)​ равен:

Именно это уравнение используют инженеры при проектировании дорог. Обратите внимание, что масса автомобиля не влияет на величину угла, при котором центростремительная сила полностью обеспечивается только горизонтальной компонентой нормальной силы. Попробуем теперь определить величину угла наклона поворота с радиусом 200 м для автомобиля, движущегося со скоростью 100 км/ч или 27,8 м/с:

Для обеспечения безопасного движения автомобиля со скоростью 100 км/ч в повороте с радиусом 200 м без учета силы трения, инженеры должны создать наклон около 22°. Отлично, из вас может получиться неплохой инженер-конструктор автомагистралей!

Вращательное движение: перемещение, скорость и ускорение

Если вы привыкли решать задачи о прямолинейном движении типа “некто движется из пункта А в пункт Б”, то задачи о вращательном движении можно формулировать аналогично, но для этого нужно приобрести некоторый опыт. На рис. 7.1 мяч движется криволинейно по окружности, а не прямолинейно по линии. Это движение можно было бы описать как комбинацию прямолинейных движений с координатами X и Y. Однако гораздо удобнее характеризовать его иначе, а именно как вращательное движение с одной координатой ​\( \theta \)​. В данном примере вращательного движения перемещение можно характеризовать углом \( \theta \) так же, как в прямолинейном движении перемещение характеризуется расстоянием \( s \). (Более подробно перемещение при прямолинейном движении описывается в главе 3.)

Стандартной единицей измерения перемещения при вращательном движении является радиан (рад), а не градус. Полная окружность охватывает угол величиной ​\( 2\pi \)​ радиан, что равно 360°. Соответственно, половина окружности охватывает угол величиной ​\( \pi \)​ радиан, а четверть окружности — ​\( \pi/2 \)​.

Как преобразуются величины углов из градусов в радианы и обратно? Достаточно определить, сколько радиан приходится на один градус, т.е. вычислить отношение ​\( 2\pi \)​/360°. Например, величина угла 45° в радианах равна:

Аналогично, для преобразования величины угла из радианов в градусы следует определить, сколько градусов приходится на один радиан, т.е. вычислить отношение 360°/​\( 2\pi \)​. Например, величина угла ​\( \pi/2 \)​ в градусах равна:

Формулировка вращательного движения в терминах прямолинейного движения очень удобна. Напомним основные формулы прямолинейного движения, которые подробно описываются в главе 3:

Теперь для вывода аналогичных основных формул вращательного движения достаточно в формулах прямолинейного движения вместо расстояния ​\( s \)​, которое характеризует прямолинейное перемещение, подставить угол ​\( \theta \)​, который характеризует угловое перемещение. А как определяется угловая скорость? Очень просто. Угловая скорость ​\( \omega \)​ определяется аналогично, как изменение угла за единицу времени, и равна количеству радианов, пройденных за секунду:

Обратите внимание, как похоже это выражение для угловой скорости на выражение для линейной скорости:

Давайте теперь вычислим угловую скорость мяча на рис. 7.1. Он совершает полный круг, охватывающий ​\( 2\pi \)​ радиан, за 1/2 с, а значит, его угловая скорость равна:

(Величина угла, выраженная в радианах, равна отношению длины дуги окружности к длине ее радиуса.{-1} \), то чему равно угловое ускорение? Подставим эти численные значения в предыдущую формулу и получим:

Итак, для описания вращательного движения у нас есть следующие аналоги: для линейного перемещения ​\( s \)​ — угловое перемещение ​\( \theta \)​, для линейной скорости ​\( v \)​ — угловая скорость ​\( \omega \)​ и для линейного ускорения ​\( a \)​ — угловое ускорение ​\( \alpha \)​.

На основании этой аналогии можно легко вывести основные формулы вращательного движения (подобно основным формулам прямолинейного движения, которые подробно описываются в главе 3):

Более подробно эти выражения рассматриваются далее в главе 10 при описании момента импульса и момента силы.

Бросаем яблоко: закон всемирного тяготения Ньютона

Чтобы проводить опыты с вращательным движением, необязательно привязывать мячики к нитям и вращать их вокруг себя. Например, Луне совсем не нужны никакие нити, чтобы вращаться вокруг Земли. А дело в том, что необходимую центростремительную силу, вместо силы натяжения нити, обеспечивает сила гравитационного притяжения.

Один из важнейших законов физики, а именно закон всемирного тяготения, вывел еще сэр Исаак Ньютон. Согласно этому закону любые два тела притягиваются друг к другу с некоторой силой. Величина этой силы притяжения между телами с массами ​\( m_1 \)​ и ​\( m_2 \)​, которые находятся на расстоянии ​\( r \)​ друг от друга, равна:

где ​\( G \)​ — это константа, равная 6,67·10-11 Н·м2/кг2.

Благодаря этому уравнению можно легко вычислить силу гравитационного притяжения между двумя телами. Например, какова сила гравитационного притяжения между Землей и Солнцем? Солнце имеет массу около 1,99·1030 кг, Земля — 5,97·1024 кг, а расстояние между ними равно 1,50·1011 м. Подставляя эти числа в закон всемирного тяготения Ньютона, получим:

Историческая яблоня

Как известно, яблоко упало на голову Исаака Ньютона, и он открыл закон всемирного тяготения. Неужели это так и было? Правда ли, что какое-то падающее яблоко натолкнуло его на верную мысль или, по крайней мере, привлекло внимание Ньютона к данной теме? Согласно последним историческим исследованиям, весьма маловероятно, что именно падение яблока на голову великого ученого вдохновило его.2 \)​ для силы гравитационного притяжения справедливо независимо от расстояния между двумя массивными телами. В обыденных ситуациях часто приходится иметь дело с небольшими (по сравнению с размерами Земли) объектами на поверхности Земли, т.е. на фиксированном расстоянии между центром Земли и центром небольшого объекта. Силу гравитационного притяжения (или силу тяжести), действующую на небольшой объект, часто называют весом. Вес ​\( F_g \)​ равен произведению массы ​\( m \)​ на ускорение свободного падения ​\( g \)​, т.е. ​\( F_g = mg \)​. Массу измеряют в граммах, килограммах, центнерах, каратах и т.д., а вес — в динах, ньютонах и даже фунт-силах.

Попробуем вычислить ускорение свободного падения на поверхности Земли, пользуясь законом всемирного тяготения. Формула веса тела с массой ​\( m_1 \)​ нам известна:

Она создается силой гравитационного притяжения между этим телом и Землей и равна этой силе:

Здесь ​\( r \)​ — это радиус Земли, равный 6,38·106 м, а ​\( m_2 \)​ — ее масса, равная 5,97·1024 кг.

Сокращая массу тела ​\( m_1 \)​ в обеих половинах предыдущего равенства, получим:

Подставляя численные значения, получим:

Так, благодаря закону всемирного тяготения Ньютона мы смогли вычислить значение ускорения свободного падения, уже известное нам из прежних глав. Как видите, для этого нам потребовались значения константы всемирного тяготения ​\( G \)​, радиуса Земли ​\( r \)​ и ее массы ​\( m_2 \)​. (Конечно, значение ускорения свободного падения ​\( g \)​ можно определить экспериментально, измеряя время падения предмета с известной высоты. Но, согласитесь, гораздо интересней использовать последнюю формулу, для применения которой потребуется экспериментально измерить… радиус и массу Земли. Шутка!)

Исследуем орбитальное движение с помощью закона всемирного тяготения

Небесные тела в космическом пространстве из-за силы гравитационного притяжения вращаются друг относительно друга: спутники — вокруг своих планет (как Луна — вокруг Земли), планеты — вокруг звезд (как Земля — вокруг Солнца в Солнечной системе), а звезды — вокруг центра Галактики (как Солнце — вокруг центра нашей галактики, т.е. Млечного пути), а Галактика — вокруг местной группы галактик (как Млечный путь — вокруг нашей Местной группы галактик). Во всех этих случаях тела удерживаются центростремительной силой, которую обеспечивает сила гравитации. Как показано ниже, такая центростремительная сила несколько отличается от той, которая известна нам по прежнему примеру с вращающимся на нитке мячом для игры в гольф. В следующих разделах рассматриваются широко известные законы вращения тел под действием силы гравитационного притяжения, так называемые законы Кеплера, т.е. соотношения между параметрами вращательного движения: периодами вращения, радиусами и площадями орбит вращения.

Вычисляем скорость спутника

Чему равна скорость спутника, вращающегося вокруг планеты по орбите с постоянным радиусом? Ее можно легко определить, приравнивая центростремительную силу:

и силу гравитации:

В итоге получаем:

После простых алгебраических операций получим следующее выражение для скорости вращения:

Это уравнение определяет скорость вращения спутника по постоянной орбите независимо от его происхождения, будь-то искусственный спутник Земли, как рукотворный космический корабль на постоянной орбите, или естественный спутник Земли, как Луна.

Подсчитаем скорость вращения искусственного спутника Земли, вращающегося вокруг Земли. Для этого нужно в предыдущую формулу подставить массу Земли и расстояние от космического орбитального спутника до центра Земли.

Рукотворные спутники Земли обычно вращаются на высоте около 640 км, а радиус Земли, как известно, равен 6,38·106 м. Можно считать, что искусственные спутники вращаются на круговой орбите с радиусом около 7,02·106 м. Подставляя это и другие известные нам численные значения в предыдущую формулу, получим:

В этом месте нужно сделать несколько важных замечаний.

Значение 7,02·106 м в знаменателе обозначает расстояние от спутника до центра Земли, а не расстояние от спутника до поверхности Земли, равное 640 км. Помните, что в законе всемирного тяготения под расстоянием между телами подразумевается расстояние между их центрами масс, а не между их поверхностями.

В данном примере предполагается, что космический корабль находится достаточно высоко и не испытывает влияние атмосферы, например силу трения от соприкосновения с ней. На самом деле это не так. Даже на такой большой высоте как 640 км, космический корабль теряет скорость, вследствие трения в разреженных слоях атмосферы. В результате его скорость уменьшается, а сам корабль постепенно снижается. (Более подробно об этом рассказывается ниже.)

Движение искусственного спутника вокруг Земли можно рассматривать как “вечное” падение. От фактического падения его “удерживает” только то, что вектор скорости всегда направлен перпендикулярно радиусу окружности вращения. Действительно, именно из-за такого “вечного” падения космонавты испытывают чувство невесомости. Дело в том, что космонавты и их космический корабль “вечно” падают по касательной к орбите вращения вокруг Земли, но при этом нисколько не приближаются к Земле.

В практических целях часто важнее знать период обращения искусственного спутника, а не его скорость. Это нужно, например, в ситуации, когда требуется определить момент выхода на связь с космическим кораблем.

Вычисляем период обращения спутника

Периодом обращения спутника называется время, которое необходимо ему, чтобы совершить полный цикл вращательного движения по орбите. Если нам известна орбитальная скорость движения ​\( v \)​ спутника по окружности с радиусом ​\( r \)​ (см. предыдущий раздел), то можно легко и просто вычислить период обращения ​\( T \)​. За период обращения спутник преодолевает расстояние, равное длине окружности ​\( 2\pi r \)​. Это значит, что орбитальная скорость ​\( v \)​ спутника равна \( 2\pi r/T \). Приравнивая это соотношение и полученное ранее выражение для орбитальной скорости

где ​\( m \)​ — масса Земли, получим:

Отсюда легко получить следующее выражение для периода обращения спутника:

А на какой высоте должен находиться спутник, чтобы вращаться с периодом обращения Земли вокруг своей оси, равным 24 часам или 86400 с? Это вовсе не праздный вопрос. Такие спутники действительно существуют и используются для обеспечения непрерывной связи в данном регионе. Действительно, ведь, обращаясь вокруг Земли с тем же периодом, что и Земля, спутник на такой геостационарной орбите постоянно находится над одной и той же точкой поверхности Земли. Несколько таких спутников образуют систему глобального позиционирования. Итак, с помощью предыдущей формулы вычислим радиус окружности вращения спутника на стационарной орбите:

Подставляя численные значения, получим:

Отнимая от этой величины 4,23·107 м, значение радиуса Земли, равное 6,38·106 м, получим приблизительно 3,59·107 м, т.е. около 35900 км. Именно на таком расстоянии от Земли вращаются спутники глобальной системы позиционирования.

На практике спутники на геостационарной орбите все же теряют скорость из- за взаимодействия с магнитным полем Земли (подробнее о магнитном поле рассказывается в следующих главах). Поэтому спутники оборудованы небольшими двигателями для корректировки их положения на геостационарной орбите.

Вращаемся вдоль вертикальной плоскости

Наверняка вам приходилось наблюдать, как отважные мотоциклисты, велосипедисты или скейтбордисты вращаются внутри круглого трека, расположенного в вертикальной плоскости. Почему сила тяжести не опрокидывает их в самой верхней точке, где они находятся вверх ногами? Как быстро им нужно двигаться, чтобы сила гравитации не превышала центростремительной силы?

Рассмотрим эту ситуацию подробнее с помощью схемы на рис. 7.4. Для простоты предположим, что вместо отважных спортсменов маленький мячик совершает движение по окружности, расположенной в вертикальной плоскости. Итак, предыдущий вопрос формулируется следующим образом: “Какой минимальной скоростью должен обладать мячик, чтобы совершить полный цикл движения по вертикально расположенной окружности?”. Какому основному условию должно отвечать движение мячика, чтобы он совершил полный цикл движения по такой окружности и не упал в самой верхней точке?

Для прохождения самой верхней точки без падения мячик должен обладать минимальной скоростью, достаточной для создания такой центростремительной силы, которая была бы не меньше силы гравитации.

При таких условиях нормальная сила со стороны трека будет равна нулю, а единственной силой, которая будет удерживать объект на окружности, является сила гравитации. Поскольку центростремительная сила равна:

а сила гравитации равна:

то, приравнивая их, получим:

Отсюда получим выражение для минимально необходимой скорости для безопасного движения по окружности, расположенной в вертикальной плоскости:

Обратите внимание, что на величину минимально необходимой скорости для безопасного движения объекта по окружности, расположенной в вертикальной плоскости, не влияет масса объекта, будь-то мячик, мотоцикл или гоночный автомобиль.

Любой объект, движущийся с меньшей скоростью, в самой верхней точке трека неизбежно отклонится от траектории движения по окружности и упадет. Давайте вычислим величину минимально необходимой скорости для безопасного движения по окружности с радиусом 20 м. Подставляя численные значения в предыдущую формулу, получим:

Итак, для безопасного движения по окружности с радиусом 20 м объект (мячик, мотоцикл или гоночный автомобиль) должен иметь скорость не менее 14 м/с, т.е. около 50 км/ч.

Учтите, что для безопасного движения по окружности такую минимальную скорость объект должен иметь в самой верхней точке! Для того чтобы развить такую скорость в верхней точке, объекту в нижней точке нужно иметь гораздо большую скорость. Действительно, ведь чтобы добраться до верхней точки объекту придется какое-то время преодолевать силу гравитации с неизбежной потерей скорости.

Возникает вопрос: какую минимальную скорость в нижней точке должен иметь объект для безопасного движения по такой окружности? Подробный ответ на этот вопрос будет дан в части III этой книги, в которой рассматриваются такие понятия, как “кинетическая энергия”, “потенциальная энергия” и “преобразование энергии из одной формы в другую”.

Глава 7. Движемся по орбитам

2.5 (49.09%) 11 votes

Открытая Физика. Вращение твердого тела

Для кинематического описания вращения твердого тела удобно использовать угловые величины: угловое перемещение Δφ, угловую скорость ω ω=ΔφΔt;  (Δt→0), и угловое ускорение ε ε=ΔωΔt;  (Δt→0).

В этих формулах углы выражаются в радианах. При вращении твердого тела относительно неподвижной оси все его точки движутся с одинаковыми угловыми скоростями и одинаковыми угловыми ускорениями. За положительное направление вращения обычно принимают направление против часовой стрелки.

Вращение диска относительно оси, проходящей через его центр O

При малых угловых перемещениях Δφ модуль вектора Δs→ линейного перемещения некоторого элемента массы Δm вращающегося твердого тела выражается соотношением: Δs = rΔφ, где r – модуль радиус-вектора r→ (рис. 1.23.1). Отсюда следует связь между модулями линейной и угловой скоростей: υ = rω, и между модулями линейного и углового ускорения: a = aτ = rε.

Векторы υ→ и a→=a→τ направлены по касательной к окружности радиуса r. Следует вспомнить, что при движении тела по окружности возникает также нормальное или центростремительное ускорение, модуль которого есть an=υ2r=ω2r.

Разобьем вращающееся тело на малые элементы Δmi. Расстояния до оси вращения обозначим через ri, модули линейных скоростей – через υi. Тогда кинетическую энергию вращающегося тела можно записать в виде: Ek=∑iΔmυi22=∑iΔm(riω)22=ω22∑iΔmiri2.

Физическая величина ∑iΔmiri2 зависит от распределения масс вращающегося тела относительно оси вращения. Она называется моментом инерции I тела относительно данной оси: I=∑iΔmiri2.

В пределе при Δm → 0 эта сумма переходит в интеграл. Единица измерения момента инерции в СИ – килограмм-метр в квадрате (кгċм2). Таким образом, кинетическую энергию твердого тела, вращающегося относительно неподвижной оси, можно представить в виде Ek=Iω22.

Эта формула очень похожа на выражение для кинетической энергии поступательно движущегося тела mυ22, только теперь вместо массы m в формулу входит момент инерции I, а вместо линейной скорости υ – угловая скорость ω.

Момент инерции в динамике вращательного движения играет ту же роль, что и масса тела в динамике поступательного движения. Но есть и принципиальная разница. Если масса – внутреннее свойство данного тела, не зависящее от его движения, то момент инерции тела зависит от того, вокруг какой оси оно вращается. Для разных осей вращения моменты инерции одного и того же тела различны.

Во многих задачах рассматривается случай, когда ось вращения твердого тела проходит через его центр массы. Положение xC, yC центра масс для простого случая системы из двух частиц с массами m1 и m2, расположенными в плоскости XY в точках с координатами x1, y1 и x2, y2 (рис. 1.23.2), определяется выражениями: xC=m1x1+m2x2m1+m2;  yC=m1y1+m2y2m1+m2.

Центр масс C системы из двух частиц

В векторной форме это соотношение принимает вид: r→C=m1r→1+m2r→2m1+m2.

Аналогично, для системы из многих частиц радиус-вектор r→C центра масс определяется выражением r→C=∑mir→i∑mi.

Для сплошного тела суммы в выражении для r→C заменяются интегралами. Легко видеть, что в однородном поле тяготения центр масс совпадает с центром тяжести. Если в однородном поле тяготения твердое тело сложной формы подвесить за центр масс, то оно будет находиться в безразличном состоянии равновесия. Поэтому положение центра масс тела сложной формы можно практически определить путем последовательного подвешивания его за несколько точек и отмечая по отвесу вертикальные линии (рис. 1.23.3).

Определение положения центра масс C тела сложной формы. A1, A2, A3 точки подвеса

Равнодействующая сил тяжести в однородном поле тяготения приложена к центру масс тела. Если тело подвешено за центр масс, то оно находится в состоянии безразличного равновесия (см. §1.14).

Любое движение твердого тела можно представить как сумму двух движений: поступательного движения со скоростью центра масс тела и вращения относительно оси, проходящей через центр масс. Примером может служить колесо, которое катится без проскальзывания по горизонтальной поверхности (рис. 1.23.4). При качении колеса все его точки движутся в плоскостях, параллельных плоскости рисунка. Такое движение называется плоским.

При плоском движении кинетическая энергия движущегося твердого тела равна сумме кинетической энергии поступательного движения и кинетической энергии вращения относительно оси, проходящей через центр масс тела и перпендикулярной плоскостям, в которых движутся все точки тела: Ek=mυC22+ICω22, где m – полная масса тела, IC – момент инерции тела относительно оси, проходящей через центр масс.

Качение колеса как сумма поступательного движения со скоростью υ→C и вращения с угловой скоростью ω=υC/R относительно оси O, проходящей через центр масс

В механике доказывается теорема о движении центра масс: под действием внешних сил центр масс любого тела или системы взаимодействующих тел движется как материальная точка, в которой сосредоточена вся масса системы.

Иллюстрацией этого утверждения может служить рис. 1.23.5, на котором изображено движение тела под действием силы тяжести. Центр масс тела движется по параболической траектории как материальная точка, в то время как все другие точки движутся по более сложным траекториям.

Движение твердого тела под действием силы тяжести

Если твердое тело вращается относительно некоторой неподвижной оси, то его момент инерции I можно выразить через момент инерции IC этого тела относительно оси, проходящей через центр масс тела и параллельной первой.

К доказательству теоремы о параллельном переносе оси вращения

Рассмотрим сечение твердого тела произвольной формы, изображенное на рис. 1.23.6. Выберем координатную систему XY с началом координат O в центре масс C тела. Пусть одна из осей вращения проходит через центр масс C, а другая через произвольную точку P, расположенную на расстоянии d от начала координат. Обе оси перпендикулярны плоскости чертежа. Пусть Δmi – некоторый малый элемент массы твердого тела. По определению момента инерции: IC=∑Δmi(xi2+yi2),IP=∑Δmi|(xi-a)2+(yi-b)2|.

Выражение для IP можно переписать в виде: IP=∑Δmi(xi2+yi2)+∑Δmi(a2+b2)-2a∑Δmixi-2b∑Δmiyi.

Поскольку начало координат совпадает с центром масс C, последние два члена обращаются в нуль. Это следует из определения центра масс. Следовательно, IP = IC + md2, где m – полная масса тела. Этот результат называют теоремой Штейнера (теоремой о параллельном переносе оси вращения).

Момент инерции

На рис. 1.23.7 изображены однородные твердые тела различной формы и указаны моменты инерции этих тел относительно оси, проходящей через центр масс.

Моменты инерции IC некоторых однородных твердых тел

Второй закон Ньютона может быть обобщен на случай вращения твердого тела относительно неподвижной оси. На рис. 1.23.8 изображено некоторое твердое тело, вращающееся относительно оси, перпендикулярной плоскости рисунка и проходящей через точку O. Выделим произвольный малый элемент массы Δmi. На него действуют внешние и внутренние силы. Равнодействующая всех сил есть F→i. Ее можно разложить на две составляющие: касательную составляющую F→iτ и радиальную F→ir. Радиальная составляющая F→ir создает центростремительное ускорение an.

Касательная F→iτ и радиальная F→ir составляющие силы F→i, действующей на элемент Δmi твердого тела

Касательная составляющая F→iτ вызывает тангенциальное ускорение a→iτ массы Δmi. Второй закон Ньютона, записанный в скалярной форме, дает Δmia = F = Fi sin θ или Δmiriε = Fi sin θ, где ε=aiτri – угловое ускорение всех точек твердого тела.

Если обе части написанного выше уравнения умножить на ri, то мы получим: Δmiri2ε=Firisinθ=Fili=Mi.

Здесь li – плечо силы F→i,  Mi – момент силы.

Теперь нужно аналогичные соотношения записать для всех элементов массы Δmi вращающегося твердого тела, а затем просуммировать левые и правые части. Это дает: ∑Δmiri2ε=∑Mi.

Стоящая в правой части сумма моментов сил, действующих на различные точки твердого тела, состоит из суммы моментов всех внешних сил и суммы моментов всех внутренних сил. ∑M=∑(Miвнешн)+∑(Miвнутр).

Но сумма моментов всех внутренних сил согласно третьему закону Ньютона равна нулю, поэтому в правой части остается только сумма моментов всех внешних сил, которые мы будем обозначать через M. В итоге: Iε = M.

Это и есть основное уравнение динамики вращательного движения твердого тела. Угловое ускорение ε и момент сил M в этом уравнении являются величинами алгебраическими. Обычно за положительное направление вращения принимают направление против часовой стрелки.

Возможна и векторная форма записи основного уравнения динамики вращательного движения, при которой величины ω→, ε→, M→ определяются как векторы, направленные по оси вращения.

При изучении поступательного движения тел вводится понятие импульса тела p→ (см. §1.16). Аналогично, при изучении вращательного движения вводится понятие момента импульса.

Моментом импульса вращающегося тела называют физическую величину, равную произведению момента инерции тела I на угловую скорость ω его вращения. Момент импульса обозначается буквой L: L = Iω.

Поскольку ε=ΔωΔt;  (Δt→0), уравнение вращательного движения можно представить в виде: M=Iε=IΔωΔt  или  MΔt=IΔω=ΔL.

Окончательно будем иметь: M=ΔLΔt;  (Δt→0).

Это уравнение, полученное здесь для случая, когда I = const, справедливо и в общем случае, когда момент инерции тела изменяется в процессе движения.

Если суммарный момент M внешних сил, действующих на тело, равен нулю, то момент импульса L = Iω относительно данной оси сохраняется: ΔL = 0, если M = 0.

Следовательно, L = Iω = const.

Это и есть закон сохранения момента импульса. Иллюстрацией этого закона может служить неупругое вращательное столкновение двух дисков, насажанных на общую ось (рис. 1.23.9).

Неупругое вращательное столкновение двух дисков. Закон сохранения момента импульса: I1ω1 = (I1 + I2

Закон сохранения момента импульса справедлив для любой замкнутой системы тел. Он выполняется, например, при движении планет по эллиптическим орбитам вокруг Солнца (второй закон Кеплера).

Уравнение вращательного движения тела можно записывать не только относительно неподвижной или равномерно движущейся оси, но и относительно оси, движущейся с ускорением.

Основное уравнение динамики вращательного движения не изменяет своего вида и в случае ускоренно движущихся осей при условии, что ось вращения проходит через центр массы тела и что ее направление в пространстве остается неизменным. Примером может служить качение тела (обруч, цилиндр, шар) по наклонной плоскости с трением (рис. 1.23.10).

Качение симметричного тела по наклонной плоскости

Ось вращения O проходит через центр масс тела. Моменты силы тяжести mg→ и силы реакции N→ относительно оси O равны нулю. Момент M создает только сила трения: M = FтрR.

Уравнение вращательного движения: ICε=ICaR=M=FтрR, где ε – угловое ускорение катящегося тела, a – линейное ускорение его центра масс, IC – момент инерции относительно оси O, проходящей через центр масс.

Второй закон Ньютона для поступательного движения центра масс записывается в виде: ma = mg sin α – Fтр.

Исключая из этих уравнений Fтр, получим окончательно: a=mgsinθ(ICR2+m).

Из этого выражения видно, что быстрее будет скатываться с наклонной плоскости тело, обладающее меньшим моментом инерции. Например, у шара IC=25mR2, а у сплошного однородного цилиндра IC=12mR2. Следовательно, шар будет скатываться быстрее цилиндра.

что это⚠️, в чем измеряется, формула для расчета

Что такое угловая скорость

​Угловая скорость (обозначается как \(\omega\)) — векторная величина, характеризующая скорость и направление изменения угла поворота со временем.

Модуль угловой скорости для вращательного движения совпадает с мгновенной угловой частотой вращения, а направление перпендикулярно плоскости вращения и связано с направлением вращения правилом правого винта.

Единица измерения

В Международной системе единиц (СИ) принятой единицей измерения угловой скорости является радиан в секунду (рад/с)

Осторожно! Если преподаватель обнаружит плагиат в работе, не избежать крупных проблем (вплоть до отчисления). Если нет возможности написать самому, закажите тут.

Формула угловой скорости

Вектор угловой скорости определяется отношением угла поворота \((\varphi)\) к интервалу времени \((\mathcal t)\), за которое произошел поворот:

\(\omega=\frac{\triangle\varphi}{\triangle\mathcal t}\)

Зависимость угловой скорости от времени

Зависимость \(\varphi \) от \(\mathcal t\) наглядно показана на графике:

 

Угол, на который повернулось тело, характеризуется площадью под кривой.

Угловая скорость вращения, формула

Через частоту

\(\omega=2\pi\mathcal n\)

\(\mathcal n\) — частота вращения \((1/с)\)

\(\pi\) — число Пи (\(\approx 3,14\))

\(\mathcal n=\frac1T\)

\(T \)— период вращения (время, за которое тело совершает один оборот)

Через радиус

\(\omega=\frac vR\)

\(v\) — линейная скорость(м/с)

\(R\) — радиус окружности (м)

Как определить направление угловой скорости

Направление скорости в физике можно определять двумя способами:

  1. Правило буравчика. Буравчик имеет правую резьбу (вращательное движение вправо при закручивании). Если вращать буравчик в направлении вращения тела, он будет завинчиваться (или вывинчиваться) в ту сторону, куда направлена угловая скорость. 
  2. Правило правой руки. Представим, что взяли тело в правую руку. Следует направлять и вращать его туда, куда указывают четыре пальца. Отведенный в сторону большой палец покажет направление угловой скорости при этом вращении.

Связь линейной и угловой скорости

Линейная скорость \((v)\) тела, расположенного на расстоянии \(R\) от оси вращения, прямо пропорциональна угловой скорости.

\(v=R\omega\)

\(R\) — радиус окружности (м)

Чему равна мгновенная угловая скорость

Мгновенную угловую скорость нужно находить как предел, к которому стремится средняя угловая скорость при \(\triangle\mathcal t\rightarrow0\) :

\(\omega=\lim_{\triangle\rightarrow0}\frac{\triangle\varphi}{\triangle\mathcal t}\)

Измеряется в рад/с

Угол вращения и угловая скорость

Физика > Вращательный угол и угловая скорость

Угол вращения – мера удаленности объекта, а угловая скорость измеряет скорость совершаемого им вращения.

Задача обучения

  • Проследите связь между вращательным углом и дистанцией.

Основные пункты

  • Когда объект совершает осевые обороты, точки на краю смещаются по дугам.
  • Угол наклона дуг именуют вращательным углом и обозначают символом тета.
  • Мера скорости вращения – угловая скорость (омега). Как и в линейной скорости, выступает вектором.

Термин

  • Радиан – угол, расположенный в центре круга по дуге окружности той же длины, что и радиус круга.

Вращательный угол и угловая скорость

Когда объект совершает осевое вращение, его движение можно описать двумя способами. Точка на его краю будет обладать определенной скоростью и проходить через дугу. Она преодолевает дистанцию ΔS, но удобнее говорить, насколько объект повернулся. Эту величину именуют углом вращения. Его можно измерить в градусах или радианах. Угол вращения связан с ΔS и радиусом в уравнении Δθ = , поэтому удобнее использовать радианы.

Радиус круга поворачивается на угол Δθ, а длина Δs описывается по окружности

Скорость вращения объекта вычисляется угловой скоростью – скорость изменения угла поворота относительно времени. Сам угол не выступает векторной величиной, но угловая скорость является ею. Направление вектора перпендикулярно плоскости вращения. Угол, угловая скорость и угловое ускорение помогают детальнее описать вращательное движение объекта.

Угловая скорость описывает скорость вращения и направление оси, вокруг которой осуществляются обороты. Направление устремляется вдоль оси. Здесь (против часовой стрелки) вектор направляется вверх

Когда ось вращения расположена перпендикулярно вектору позиции, угловую скорость можно рассчитать при помощи линейной скорости (v) на краю вращающегося объекта и разделения на радиус. Это поможет вычислить угловую скорость (ω) в радианах в секунду.

Муха на краю вращающегося тела отображает постоянную скорость (v). Угловая скорость объекта равняется


Презентация по физике » Движение по окружности»

Равномерное движение по окружности решение Задач

Типовые задачи по теме:

1. Колесо делает 120 оборотов за 2 минуты. Какова частота вращения колеса и период вращения?

2. Шарик вращают на нитке длиной 0,5 м так, что он делает за одну секунду 3 оборота. С какой линейной и угловой скоростью движется шарик.

3. Линейная скорость точек вращающегося колеса 20 м/сек. Определите их угловую скорость движения, период и частоту вращения, если диаметр колеса 0,8 метра.

4. Велосипедист движется со скоростью 36 км/час. Определите частоту вращения велосипедного колеса, имеющего диаметр 0,6 метра, период его вращения, угловую и линейную скорости точек колеса относительно оси его вращения.

Краткая теория:

Равномерное движение по окружности интересно тем, что скорость движущейся точки остается постоянной по величине, изменяясь при этом по направлению. Скорость изменения угла вектора скорости относительно оси координат постоянна. То же самое можно сказать относительно радиуса-вектора, проведенного из оси вращения к вращающейся точке. Эта скорость называется угловой скоростью.

Равномерное движение по окружности характеризуется несколькими взаимосвязанными величинами:

Частота вращения. Обычно обозначается латинской буквой «n» или греческой буквой «?». Эта величина говорит о том, сколько оборотов в единицу времени делает тело. Например, сколько оборотов в секунду, или в минуту, или в час и т.д.

Период вращения чаще всего обозначается латинской буквой «T». Это время одного оборота вокруг оси.

Линейная скорость вращения, обозначается обычно латинской буквой «v». Это скорость, с которой тело движется по окружности. Вектор линейной скорости направлен по касательной к окружности вращения. Он перпендикулярен радиусу окружности вращения.

Угловая скорость вращения обычно обозначается греческой буквой «?». Это величина, показывающая, на какой угол поворачивается радиус-вектор (или вектор скорости) за единицу времени. Обычно измеряется в радианах в секунду.

Формулы для решения:

Частота вращения.

Где N — количество оборотов, t — время, за которое они совершились.

Период вращения

Линейная скорость вращения

Угловая скорость вращения

Задача 1.

Колесо делает 120 оборотов за 2 минуты. Какова частота вращения колеса и период вращения?

Решение.

Решаем по алгоритму.

1. Кратко записываем условие задачи.

2. Изображаем графически движение, нарисовав вращающееся колесо и обозначив стрелкой направление вращения.

3. Систему отсчета в явном виде можно не вводить. В неявном виде она, конечно же присутствует, поскольку мы должны произвести отсчет времени и оборотов.

4. Записываем необходимые для решения формулы.

5. Эти уравнения сразу дают нам результат в общем виде.

6. Подставляем заданные величины в общее решение, вычисляем.

Переводя в систему единиц СИ, получаем: 60 об/мин=1 об/сек, 1/60 мин=1 сек.

7. Записываем ответ.

Ответ: Частота вращения колеса 1 оборот в секунду, период вращения 1 секунда.

Задача 2.

Шарик вращают на нитке длиной 0,5 м так, что он делает за одну секунду 3 оборота. С какой линейной и угловой скоростью движется шарик.

Решение.

1,2. Кратко записываем условие задачи, изображая рядом движение.

3. Вводим систему отсчета, начав отсчет времени в момент нахождения шарика в нижней точке и разместив начало системы координат на шарике, направив одну ось вдоль радиуса, а вторую вдоль скорости.

4. Записываем необходимые для решения формулы.

5. Записанные формулы сразу дают решение в общем виде.

6. Подставляем заданные величины в общее решение, вычисляем.

7. Записываем ответ.

Ответ: Скорость движения шарика по окружности 9,42 м/сек, угловая скорость — 18,84 рад/сек.

Задача 3.

Линейная скорость точек вращающегося колеса 20 м/сек. Определите их угловую скорость движения, период и частоту вращения, если диаметр колеса 0,8 метра.

Решение.

Решаем по алгоритму.

  • Кратко записываем условие задачи.

2. Изображаем графически движение колеса, обозначаем стрелками скорость и направление вращения.

3. Вводим систему отсчета, связав отсчета времени и ноль координат с нижней точкой колеса, направив одну ось вдоль радиуса, тогда вторая ось будет направлена вдоль скорости.

5. Решаем эти уравнения в общем виде.

4. Записываем необходимые для решения формулы.

6. Подставляем заданные величины, вычисляем.

7. Записываем ответ.

Ответ: Угловая скорость движения точек колеса 50 радиан в секунду, частота вращения 80 оборотов в секунду, период вращения 125 десятитысячных секунды.

Задача 4

Велосипедист движется со скоростью 36 км/час. Определите частоту вращения велосипедного колеса, имеющего диаметр 0,6 метра, период его вращения, угловую и линейную скорости точек колеса относительно оси его вращения.

Решение.

Решаем по алгоритму.

1. Кратко записываем условие задачи.

2. Изображаем графически движение, нарисовав окружность вращения и обозначив стрелками скорость и направление движения.

3. Введем систему отсчета. Выберем среди равноправных точек колеса ту, которая в момент начала отсчета времени касалась земли. Начало оси координат поместим в точку их первого (по нашему отсчету) соприкосновения.

4. Запишем необходимые для решения формулы, для чего сначала проанализируем движение велосипеда и движение точек колеса. В этом движении колесо прокатится на один оборот и замеченная нами точка вновь окажется внизу, а ось опять точно над ней. Но время одного оборота — это же период вращения колеса! То есть время, за которое будет пройден путь, равный длине окружности колеса — это период его вращения. Это время легко найти, зная путь и скорость.

Обозначим длину окружности колеса через «s», время прохождения этого пути через «t», искомый период вращения через «T». Выше мы выяснили, что

Если мы знаем период и радиус колеса, то легко найти все остальное из следующих уравнений.

5. Решаем уравнения в общем виде.

6. Подставляем заданные значения, вычисляем. Величины должны быть измерены в одних единицах. Переводим километры в час в метры в секунду. В одном километре 1000 метров, а в одном часе 3600 секунд.

7. Записываем ответ.

Ответ: Период обращения колеса велосипеда 19 сотых секунды, частота вращения 5,25 оборота в секунду, угловая скорость 33,3 радиана в секунду, линейная скорость точек колеса 10 метров в секунду.

Угловая скорость | Медицинские журналы

В физике угловая скорость относится к тому, насколько быстро объект вращается или вращается относительно другой точки, то есть насколько быстро угловое положение или ориентация объекта изменяется со временем. Есть два типа угловой скорости: орбитальная угловая скорость и угловая скорость вращения. Угловая скорость вращения означает, насколько быстро твердое тело вращается относительно центра вращения. Орбитальная угловая скорость относится к тому, насколько быстро точечный объект вращается вокруг фиксированной точки начала координат, т.е.е. скорость изменения его углового положения относительно начала координат. Угловая скорость вращения не зависит от выбора начала координат, в отличие от орбитальной угловой скорости, которая зависит от выбора начала координат.

Обычно угловая скорость измеряется в углах в единицу времени, например радиан в секунду (угол, заменяющий расстояние от линейной скорости от времени в общем). Единица измерения угловой скорости в системе СИ выражается в радианах в секунду, причем радиан имеет безразмерное значение, равное единице, поэтому единицы измерения угловой скорости в системе СИ обозначаются как 1 / с или с − 1.Угловая скорость обычно обозначается символом омега (ω, иногда Ω). По соглашению, положительная угловая скорость означает вращение против часовой стрелки, а отрицательная — по часовой стрелке.

Например, геостационарный спутник совершает один оборот в день над экватором, или 360 градусов за 24 часа, и имеет угловую скорость ω = (360 °) / (24 ч) = 15 ° / ч, или (2π рад) / (24 ч) ≈ 0,26 рад / ч. Если угол измеряется в радианах, линейная скорость равна радиусу, умноженному на угловую скорость, {\ displaystyle v = r \ omega} {\ displaystyle v = r \ omega}.Таким образом, с радиусом орбиты 42000 км от центра Земли скорость спутника в космосе составляет v = 42000 км × 0,26 / ч ≈ 11000 км / ч. Угловая скорость положительна, поскольку спутник движется на восток вместе с вращением Земли (против часовой стрелки от северного полюса). В трех измерениях угловая скорость является псевдовектором, величина которого измеряет скорость вращения или вращения объекта, и его направление перпендикулярно плоскости мгновенного вращения или углового смещения.Ориентация угловой скорости условно задается правилом правой руки.

В физике угловая скорость относится к тому, насколько быстро объект вращается или вращается относительно другой точки, то есть насколько быстро угловое положение или ориентация объекта изменяется со временем. Есть два типа угловой скорости: орбитальная угловая скорость и угловая скорость вращения. Угловая скорость вращения означает, насколько быстро твердое тело вращается относительно центра вращения. Орбитальная угловая скорость относится к тому, насколько быстро точечный объект вращается вокруг фиксированной точки начала координат, т.е.е. скорость изменения его углового положения относительно начала координат. Угловая скорость вращения не зависит от выбора начала координат, в отличие от орбитальной угловой скорости, которая зависит от выбора начала координат.

Обычно угловая скорость измеряется в углах в единицу времени, например радиан в секунду (угол, заменяющий расстояние от линейной скорости от времени в общем). Единица измерения угловой скорости в системе СИ выражается в радианах в секунду, причем радиан имеет безразмерное значение, равное единице, поэтому единицы измерения угловой скорости в системе СИ обозначаются как 1 / с или с − 1.Угловая скорость обычно обозначается символом омега (ω, иногда Ω). По соглашению, положительная угловая скорость означает вращение против часовой стрелки, а отрицательная — по часовой стрелке.

Например, геостационарный спутник совершает один оборот в день над экватором, или 360 градусов за 24 часа, и имеет угловую скорость ω = (360 °) / (24 ч) = 15 ° / ч, или (2π рад) / (24 ч) ≈ 0,26 рад / ч. Если угол измеряется в радианах, линейная скорость равна радиусу, умноженному на угловую скорость, {\ displaystyle v = r \ omega} {\ displaystyle v = r \ omega}.Таким образом, с радиусом орбиты 42000 км от центра Земли скорость спутника в космосе составляет v = 42000 км × 0,26 / ч ≈ 11000 км / ч. Угловая скорость положительна, поскольку спутник движется на восток вместе с вращением Земли (против часовой стрелки от северного полюса). В трех измерениях угловая скорость является псевдовектором, величина которого измеряет скорость вращения или вращения объекта, и его направление перпендикулярно плоскости мгновенного вращения или углового смещения.Ориентация угловой скорости условно задается правилом правой руки.

Актуальные темы общих наук

10.2: Вращательные переменные — Physics LibreTexts

До сих пор в этом тексте мы в основном изучали поступательное движение, включая переменные, которые его описывают: смещение, скорость и ускорение.Теперь мы расширим наше описание движения до вращения, в частности, вращательного движения вокруг фиксированной оси. Мы обнаружим, что вращательное движение описывается набором связанных переменных, аналогичных тем, которые мы использовали для поступательного движения.

Угловая скорость

Равномерное круговое движение (обсуждавшееся ранее в разделе «Движение в двух и трех измерениях») — это движение по кругу с постоянной скоростью. Хотя это простейший случай вращательного движения, он очень полезен во многих ситуациях, и мы используем его здесь, чтобы ввести вращательные переменные.

На рисунке \ (\ PageIndex {1} \) мы показываем частицу, движущуюся по кругу. Система координат фиксирована и служит точкой отсчета для определения положения частицы. Его вектор положения от начала круга до частицы сметает угол \ (\ theta \), который увеличивается в направлении против часовой стрелки, когда частица движется по круговой траектории. Угол \ (\ theta \) называется угловым положением частицы. Когда частица движется по круговой траектории, она также отслеживает длину дуги s.

Рисунок \ (\ PageIndex {1} \): частица движется по круговой траектории. При движении против часовой стрелки он выметает положительный угол \ (\ theta \) по отношению к оси x и очерчивает дугу длиной s.

Угол связан с радиусом окружности и длиной дуги на

.

\ [\ theta = \ frac {s} {r} \ ldotp \ label {10.1} \]

Угол \ (\ theta \), угловое положение частицы на ее пути, измеряется в радианах (рад). В 360 ° есть \ (2 \ pi \) радианы. Обратите внимание, что мера в радианах представляет собой отношение измерений длины и, следовательно, является безразмерной величиной.По мере того, как частица движется по круговой траектории, ее угловое положение изменяется, и она претерпевает угловые смещения \ (\ Delta \ theta \).

Мы можем назначить векторы для величин в уравнении \ ref {10.1}. Угол \ (\ vec {\ theta} \) — это вектор вне страницы на рисунке \ (\ PageIndex {1} \). Вектор углового положения \ (\ vec {r} \) и длина дуги \ (\ vec {s} \) лежат в плоскости страницы. Эти три вектора связаны между собой соотношением

.

\ [\ vec {s} = \ vec {\ theta} \ times \ vec {r} \ ldotp \ label {10.2} \]

То есть длина дуги — это произведение вектора угла и вектора положения, как показано на рисунке \ (\ PageIndex {2} \).

Рисунок \ (\ PageIndex {2} \): точки вектора угла вдоль оси z, вектор положения и вектор длины дуги лежат в плоскости xy. Мы видим, что \ (\ vec {s} = \ vec {\ theta} \ times \ vec {r} \). Все три вектора перпендикулярны друг другу.

Величина угловой скорости, обозначаемая \ (\ omega \), представляет собой скорость изменения угла \ (\ theta \) во времени, когда частица движется по круговой траектории.Мгновенная угловая скорость определяется как предел, при котором \ (\ Delta \) t → 0 в средней угловой скорости \ (\ bar {\ omega} = \ frac {\ Delta \ theta} {\ Delta t} \):

\ [\ omega = \ lim _ {\ Delta t \ rightarrow 0} \ frac {\ Delta \ theta} {\ Delta t} = \ frac {d \ theta} {dt}, \ label {10.3} \]

, где \ (\ theta \) — угол поворота (Рисунок \ (\ PageIndex {2} \)). Единицы угловой скорости — радианы в секунду (рад / с). Угловая скорость также может называться скоростью вращения в радианах в секунду.Во многих ситуациях нам задают скорость вращения в оборотах в секунду или циклах в секунду. Чтобы найти угловую скорость, мы должны умножить число оборотов в секунду на 2 \ (\ pi \), поскольку за один полный оборот приходится 2 \ (\ pi \) радиан. Поскольку направление положительного угла в круге — против часовой стрелки, мы принимаем вращения против часовой стрелки как положительные, а вращения по часовой стрелке — как отрицательные.

Мы можем увидеть, как угловая скорость связана с тангенциальной скоростью частицы, дифференцируя уравнение \ ref {10.1} по времени. Мы перепишем уравнение \ ref {10.1} как

\ [s = r \ theta \ ldotp \]

Взяв производную по времени и отметив, что радиус r является константой, получаем

\ [\ frac {ds} {dt} = \ frac {d} {dt} (r \ theta) = \ theta \ frac {dr} {dt} + r \ frac {d \ theta} {dt} = r \ frac {d \ theta} {dt} \]

, где \ (\ theta \ frac {dr} {dt} \) = 0. Здесь \ (\ frac {ds} {dt} \) — это просто тангенциальная скорость v t частицы на рисунке \ (\ PageIndex {1} \).Таким образом, используя уравнение \ ref {10.3}, мы получаем

\ [v_ {t} = r \ omega \ ldotp \ label {10.4} \]

То есть тангенциальная скорость частицы равна ее угловой скорости, умноженной на радиус круга. Из уравнения \ ref {10.4} видно, что тангенциальная скорость частицы увеличивается по мере удаления от оси вращения для постоянной угловой скорости. Этот эффект показан на рисунке \ (\ PageIndex {3} \). Две частицы размещены под разными радиусами на вращающемся диске с постоянной угловой скоростью.По мере вращения диска тангенциальная скорость увеличивается линейно с радиусом от оси вращения. На рисунке \ (\ PageIndex {3} \) мы видим, что v 1 = r 1 \ (\ omega_ {1} \) и v 2 = r 2 \ (\ omega_ {2} \). Но диск имеет постоянную угловую скорость, поэтому \ (\ omega_ {1} = \ omega_ {2} \). Это означает \ (\ frac {v_ {1}} {r_ {1}} = \ frac {v_ {2}} {r_ {2}} \) или v 2 = \ (\ left (\ dfrac {r_ {2}} {r_ {1}} \ right) \) v 1 . Таким образом, поскольку r 2 > r 1 , v 2 > v 1 .

Рисунок \ (\ PageIndex {3} \): Две частицы на вращающемся диске имеют разные тангенциальные скорости в зависимости от их расстояния до оси вращения.

До сих пор мы обсуждали величину угловой скорости \ (\ omega = \ frac {d \ theta} {dt} \), которая является скалярной величиной — изменением углового положения во времени. Вектор \ (\ vec {\ omega} \) — это вектор, связанный с угловой скоростью и направленный вдоль оси вращения. Это полезно, потому что, когда твердое тело вращается, мы хотим знать как ось вращения, так и направление, в котором тело вращается вокруг оси, по или против часовой стрелки.Угловая скорость \ (\ vec {\ omega} \) дает нам эту информацию. Угловая скорость \ (\ vec {\ omega} \) имеет направление, определяемое так называемым правилом правой руки. Правило правой руки таково, что если пальцы вашей правой руки повернутся против часовой стрелки от оси x (направление, в котором \ (\ theta \) увеличивается) к оси y, ваш большой палец будет указывать в направлении положительного Ось z (рисунок \ (\ PageIndex {4} \)). Следовательно, угловая скорость \ (\ vec {\ omega} \), которая указывает вдоль положительной оси z, соответствует вращению против часовой стрелки, тогда как угловая скорость \ (\ vec {\ omega} \), которая указывает вдоль отрицательной оси z соответствует вращению по часовой стрелке.

Рисунок \ (\ PageIndex {4} \): Для вращения против часовой стрелки в показанной системе координат угловая скорость указывает в положительном направлении z по правилу правой руки.

Мы можем проверить правило правой руки, используя векторное выражение для длины дуги \ (\ vec {s} = \ vec {\ theta} \ times \ vec {r} \), уравнение \ ref {10.2}. Если продифференцировать это уравнение по времени, находим

\ [\ frac {d \ vec {s}} {dt} = \ frac {d} {dt} (\ vec {\ theta} \ times \ vec {r}) = \ left (\ dfrac {d \ theta } {dt} \ times \ vec {r} \ right) + \ left (\ vec {\ theta} \ times \ dfrac {d \ vec {r}} {dt} \ right) = \ frac {d \ theta} {dt} \ times \ vec {r} \ ldotp \]

Поскольку \ (\ vec {r} \) константа, член \ (\ vec {\ theta} \ times \ frac {d \ vec {r}} {dt} \) = 0.Поскольку \ (\ vec {v} = \ frac {d \ vec {s}} {dt} \) — тангенциальная скорость, а \ (\ omega = \ frac {d \ vec {\ theta}} {dt} \) — угловая скорость, имеем

\ [\ vec {v} = \ vec {\ omega} \ times \ vec {r} \ ldotp \ label {10.5} \]

То есть тангенциальная скорость — это векторное произведение угловой скорости и вектора положения, как показано на рисунке \ (\ PageIndex {5} \). Из части (а) этого рисунка мы видим, что при угловой скорости в положительном направлении оси z вращение в плоскости xy происходит против часовой стрелки.В части (b) угловая скорость находится в отрицательном направлении z, что дает вращение по часовой стрелке в плоскости xy.

Рисунок \ (\ PageIndex {5} \): Показанные векторы — это угловая скорость, положение и тангенциальная скорость. (а) Угловая скорость указывает в положительном направлении оси z, что дает вращение против часовой стрелки в плоскости xy. (b) Угловая скорость указывает в отрицательном направлении оси z, что дает вращение по часовой стрелке.

Пример \ (\ PageIndex {1} \): вращение маховика

Маховик вращается так, что охватывает угол со скоростью \ (\ theta \) = \ (\ omega \) t = (45.0 рад / с) t радиан. Колесо вращается против часовой стрелки, если смотреть в плоскости страницы. а) Какова угловая скорость маховика? (б) В каком направлении угловая скорость? (c) На сколько радиан проходит маховик за 30 с? (d) Какова тангенциальная скорость точки на маховике на расстоянии 10 см от оси вращения?

Стратегия

Функциональная форма углового положения маховика задается в задаче как \ (\ theta \) (t) = \ (\ omega \) t, поэтому, взяв производную по времени, мы можем найти угловой скорость.Мы используем правило правой руки, чтобы найти угловую скорость. Чтобы найти угловое смещение маховика за 30 с, мы ищем угловое смещение \ (\ Delta \ theta \), где изменение углового положения составляет от 0 до 30 с. Чтобы найти тангенциальную скорость точки, находящейся на расстоянии от оси вращения, мы умножаем ее расстояние на угловую скорость маховика.

Решение

  1. \ (\ omega \) = \ (\ frac {d \ theta} {dt} \) = 45 рад / с. Мы видим, что угловая скорость постоянна.
  2. По правилу правой руки мы сгибаем пальцы в направлении вращения, которое находится против часовой стрелки в плоскости страницы, а большой палец указывает в направлении угловой скорости, которая выходит за пределы страницы.
  3. \ (\ Delta \ theta \) = \ (\ theta \) (30 с) — \ (\ theta \) (0 с) = 45,0 (30,0 с) — 45,0 (0 с) = 1350,0 рад.
  4. v t = r \ (\ omega \) = (0,1 м) (45,0 рад / с) = 4,5 м / с.

Значение

За 30 с маховик совершил большое количество оборотов, около 215, если разделить угловое смещение на 2 \ (\ pi \).Таким образом можно использовать массивный маховик для хранения энергии, если потери на трение минимальны. В недавних исследованиях рассматривались сверхпроводящие подшипники, на которых покоится маховик, с нулевыми потерями энергии из-за трения.

Угловое ускорение

Мы только что обсудили угловую скорость для равномерного кругового движения, но не все движения однородны. Представьте себе фигуриста, вращающегося с вытянутыми руками — когда он втягивает руки внутрь, его угловая скорость увеличивается.Или представьте себе, как жесткий диск компьютера останавливается при уменьшении угловой скорости. Мы рассмотрим эти ситуации позже, но мы уже видим необходимость определения углового ускорения для описания ситуаций, когда \ (\ omega \) изменяется. Чем быстрее изменяется \ (\ omega \), тем больше угловое ускорение. Мы определяем мгновенное угловое ускорение \ (\ alpha \) как производную угловой скорости по времени:

\ [\ alpha = \ lim _ {\ Delta t \ rightarrow 0} \ frac {\ Delta \ omega} {\ Delta t} = \ frac {d \ omega} {dt} = \ frac {d ^ {2} \ тета} {дт ^ {2}}, \ label {10.6} \]

, где мы взяли предел среднего углового ускорения \ (\ bar {\ alpha} = \ frac {\ Delta \ omega} {\ Delta t} \) как \ (\ Delta t → 0 \). Единицы углового ускорения: (рад / с) / с или рад / с 2 .

Точно так же, как мы определили вектор, связанный с угловой скоростью \ (\ vec {\ omega} \), мы можем определить \ (\ vec {\ alpha} \), вектор, связанный с угловым ускорением (рисунок \ (\ PageIndex {6} \)). Если угловая скорость направлена ​​вдоль положительной оси z, как на рисунке \ (\ PageIndex {4} \), и \ (\ frac {d \ omega} {dt} \) положительна, то угловое ускорение \ (\ vec {\ alpha} \) положительно и направлено вдоль оси + z-.Аналогично, если угловая скорость \ (\ vec {\ omega} \) находится вдоль положительной оси z, а \ (\ frac {d \ omega} {dt} \) отрицательна, угловое ускорение отрицательно и указывает вдоль ось + z.

Рисунок \ (\ PageIndex {6} \): Вращение против часовой стрелки в (a) и (b) с угловой скоростью в одном направлении. (а) Угловое ускорение совпадает с направлением угловой скорости, что увеличивает скорость вращения. (b) Угловое ускорение противоположно угловой скорости, что снижает скорость вращения.

Мы можем выразить вектор тангенциального ускорения как произведение углового ускорения и вектора положения. Это выражение можно найти, взяв производную по времени от \ (\ vec {v} = \ vec {\ omega} \ times \ vec {r} \), и оставим его в качестве упражнения:

\ [\ vec {a} = \ vec {\ alpha} \ times \ vec {r} \ ldotp \ label {10.7} \]

Векторные соотношения для углового и тангенциального ускорений показаны на рисунке \ (\ PageIndex {7} \).

Рисунок \ (\ PageIndex {7} \): (a) Угловое ускорение — это положительное направление по оси z, которое создает тангенциальное ускорение против часовой стрелки.(b) Угловое ускорение имеет отрицательное значение по оси z и вызывает тангенциальное ускорение по часовой стрелке.

Мы можем связать тангенциальное ускорение точки вращающегося тела на расстоянии от оси вращения таким же образом, как мы связали тангенциальную скорость с угловой скоростью. Если мы продифференцируем уравнение \ ref {10.4} по времени, отметив, что радиус r постоянен, мы получим

\ [a_ {t} = r \ alpha \ ldotp \ label {10.8} \]

Таким образом, тангенциальное ускорение a t равно радиусу, умноженному на угловое ускорение.Уравнения \ ref {10.4} и \ ref {10.8} важны для обсуждения качения (см. Угловой момент).

Давайте применим эти идеи к анализу нескольких простых сценариев вращения с фиксированной осью. Прежде чем сделать это, мы представляем стратегию решения проблем, которая может быть применена к вращательной кинематике: описание вращательного движения.

Стратегия решения проблем: вращательная кинематика

  1. Изучите ситуацию, чтобы определить, задействована ли кинематика вращения (вращательное движение).
  2. Определите, что именно необходимо определить в проблеме (определите неизвестные). Набросок ситуации полезен.
  3. Составьте полный список того, что дано или может быть выведено из указанной проблемы (укажите известные).
  4. Решите соответствующее уравнение или уравнения для величины, которую необходимо определить (неизвестное). Может быть полезно думать в терминах поступательного аналога, потому что теперь вы знакомы с уравнениями поступательного движения.
  5. Подставьте известные значения вместе с их единицами измерения в соответствующее уравнение и получите численные решения с указанием единиц. Обязательно используйте радианы для углов.
  6. Проверьте свой ответ, чтобы узнать, разумен ли он: имеет ли ваш ответ смысл?

Теперь давайте применим эту стратегию решения проблем к нескольким конкретным примерам.

Пример \ (\ PageIndex {2} \): вращающееся велосипедное колесо

Веломеханик устанавливает велосипед на ремонтный стенд и запускает вращение заднего колеса с момента покоя до конечной угловой скорости 250 об / мин за 5 секунд.00 с. (a) Рассчитайте среднее угловое ускорение в рад / с 2 . (b) Если теперь она нажимает на тормоз, вызывая угловое ускорение -87,3 рад / с 2 , сколько времени потребуется колесу, чтобы остановиться?

Стратегия

Среднее угловое ускорение можно найти непосредственно из его определения \ (\ bar {\ alpha} = \ frac {\ Delta \ omega} {\ Delta t} \), поскольку даны окончательная угловая скорость и время. Мы видим, что \ (\ Delta \ omega \) = \ (\ omega_ {final} \) — \ (\ omega_ {initial} \) = 250 об / мин и \ (\ Delta \) t равно 5.00 с. Для части (b) нам известны угловое ускорение и начальная угловая скорость. Мы можем найти время остановки, используя определение среднего углового ускорения и решение для \ (\ Delta \) t, что дает

\ [\ Delta t = \ frac {\ Delta \ omega} {\ alpha} \ ldotp \]

Решение

  1. Вводя известную информацию в определение углового ускорения, мы получаем $$ \ bar {\ alpha} = \ frac {\ Delta \ omega} {\ Delta t} = \ frac {250 \; об / мин} {5.00 \; s} \ ldotp $$ Поскольку \ (\ Delta \ omega \) выражается в оборотах в минуту (об / мин), и нам нужны стандартные единицы рад / с 2 для углового ускорения, нам нужно преобразовать из об / мин в рад / с : $$ \ Delta \ omega = 250 \ frac {rev} {min} \; \ cdotp \ frac {2 \ pi \; рад} {рев} \; \ cdotp \ frac {1 \; min} {60 \; s} = 26.{2}} = 0,300 \; s \ ldotp $$

Значение

Обратите внимание, что угловое ускорение, когда механик раскручивает колесо, небольшое и положительное; для получения заметной угловой скорости требуется 5 с. Когда она нажимает на тормоз, угловое ускорение велико и отрицательно. Угловая скорость быстро стремится к нулю.

Упражнение \ (\ PageIndex {1} \)

Лопасти вентилятора турбореактивного реактивного двигателя (показан ниже) ускоряются от состояния покоя до частоты вращения 40.0 об / с за 20 с. Увеличение угловой скорости вентилятора постоянно во времени. (Турбореактивный двигатель GE90-110B1, установленный на Боинг 777, как показано, в настоящее время является самым большим турбовентиляторным двигателем в мире, способным развивать тягу 330–510 кН.) (A) Какое среднее угловое ускорение? (б) Каково мгновенное угловое ускорение в любой момент в течение первых 20 с?

Пример \ (\ PageIndex {3} \): Ветряная турбина

Ветряная турбина (рис. \ (\ PageIndex {9} \)) в ветряной электростанции останавливается на техническое обслуживание.{2}} {100.0} \ Big] \) рад / с. Если турбина вращается против часовой стрелки, глядя на страницу, (а) каковы направления векторов угловой скорости и ускорения? б) Что такое среднее угловое ускорение? (c) Какое мгновенное угловое ускорение при t = 0,0, 15,0, 30,0 с?

Рисунок \ (\ PageIndex {9} \): ветряная турбина, вращающаяся против часовой стрелки, если смотреть спереди.

Стратегия

  1. Нам дано направление вращения турбины против часовой стрелки в плоскости страницы.Используя правило правой руки (рис. 10.5), мы можем установить направления векторов угловой скорости и ускорения.
  2. Рассчитываем начальную и конечную угловые скорости, чтобы получить среднее угловое ускорение. Мы устанавливаем знак углового ускорения из результатов в (а).
  3. Нам дана функциональная форма угловой скорости, поэтому мы можем найти функциональную форму функции углового ускорения, взяв ее производную по времени.

Решение

  1. Поскольку турбина вращается против часовой стрелки, угловая скорость \ (\ vec {\ omega} \) указывает за пределы страницы. Но поскольку угловая скорость уменьшается, угловое ускорение \ (\ vec {\ alpha} \) указывает на страницу в противоположном смысле угловой скорости.
  2. Начальная угловая скорость турбины при t = 0 равна \ (\ omega \) = 9,0 рад / с. Конечная угловая скорость равна нулю, поэтому среднее угловое ускорение составляет $$ \ bar {\ alpha} \ frac {\ Delta \ omega} {\ Delta t} = \ frac {\ omega — \ omega_ {0}} {t — t_ {0}} = \ frac {0–9.{2} и \; \ альфа (30,0 \; s) = 0 \; рад / с \ ldotp $$

Значение

Мы обнаружили из расчетов в (a) и (b), что угловое ускорение α и среднее угловое ускорение \ (\ bar {\ alpha} \) отрицательны. Турбина имеет угловое ускорение, противоположное ее угловой скорости.

Теперь у нас есть базовый словарь для обсуждения кинематики вращения с фиксированной осью и взаимосвязей между переменными вращения. Мы обсудим больше определений и связей в следующем разделе.

Угловая скорость

— обзор

3.09.4.1.1 Вековая тенденция, приливная диссипация и ледниковая изостатическая корректировка

Приливная диссипация вызывает уменьшение угловой скорости Земли и, следовательно, углового момента вращения ( см. Глава 3.07). Поскольку угловой момент системы Земля-Луна сохраняется, орбитальный угловой момент Луны должен увеличиваться, чтобы уравновесить уменьшение углового момента вращения Земли. Увеличение орбитального углового момента Луны достигается за счет увеличения радиуса орбиты Луны и уменьшения орбитальной угловой скорости Луны.Но ранние наблюдения за положением Луны показали, что она явно ускоряется, а не замедляется на своей орбите. Это кажущееся ускорение Луны было результатом предположения, что Земля вращается с постоянной, а не уменьшающейся угловой скоростью при прогнозировании положения Луны. Если угловая скорость Земли на самом деле уменьшается, но считается постоянной при прогнозировании положения Луны, то наблюдаемое положение Луны будет опережать ее прогнозируемое положение, то есть Луна будет казаться ускоряющейся в его орбита.То, что Луна явно ускоряется по своей орбите, было впервые отмечено Галлеем (1695). Но только в 1939 году Спенсер Джонс (1939) смог убедительно продемонстрировать, что угловая скорость Земли на самом деле уменьшается и что видимое ускорение Луны на ее орбите было артефактом предположения, что угловая скорость Земли был постоянным.

Галлей (1695) также, кажется, был первым, кто оценил важность древних и средневековых записей о лунных и солнечных затмениях для определения видимого ускорения Луны и соответствующего уменьшения угловой скорости Земли за последние несколько лет. тысяча лет.Изменение скорости вращения Земли можно вывести из несоответствия между тем, когда и где должны были наблюдаться затмения, если бы угловая скорость Земли была постоянной, а когда и где они действительно наблюдались, как записано на вавилонских глиняных табличках и на китайских, европейских и арабские книги и рукописи (Stephenson, 1997).

При использовании наблюдений за затмениями для определения векового изменения продолжительности дня за последние несколько тысяч лет необходимо точно знать положение Солнца и Луны.В этом отношении первостепенное значение имеет значение приливного ускорения n ̇ Луны, поскольку оно определяет долгосрочное поведение Луны. Приливное ускорение Луны может быть определено из наблюдений за временем прохождения Меркурия (например, Spencer Jones, 1939; Morrison and Ward, 1975), а также из спутниковых и лунных лазерных измерений. Приливные силы искажают фигуру Земли и, следовательно, ее гравитационное поле, которое, в свою очередь, нарушает орбиты искусственных спутников.Измерения SLR могут обнаруживать эти приливные возмущения на орбитах спутников и, следовательно, могут использоваться для построения моделей приливов и, следовательно, определения приливного ускорения Луны. Используя этот подход, Christodoulidis et al. (1988) сообщает о величине -25,27 ± 0,61 угловой секунды в столетие 2 (″ / cy 2 ) для приливного ускорения n ̇ Луны из-за рассеяния твердой Землей и океанских приливов. Другие полученные из SLR значения для n ̇ были сообщены Cheng et al. (1990, 1992), Marsh et al. (1990, 1991), Dickman (1994), Lerch et al. (1994) и Рэй (1994).

Подобно орбитам искусственных спутников, орбита Луны также подвержена влиянию приливных сил. Поскольку измерения LLR могут обнаруживать приливные возмущения на орбите Луны, их можно использовать для определения приливного ускорения Луны. Помимо чувствительности к орбитальным возмущениям, вызванным приливами на Земле, измерения LLR, в отличие от измерений SLR, также чувствительны к орбитальным возмущениям, вызванным приливами на Луне.Используя измерения LLR, Williams et al. (2001) сообщает о значении -25,73 ± 0,5 ″ / cy 2 для приливного ускорения Луны, что по закону Кеплера соответствует увеличению на 3,79 ± 0,07 см в год −1 в большой полуоси Луны. Орбита Луны, включая вклад в +0.29 ″ / cy 2 от рассеяния внутри самой Луны. В настоящее время изучена лишь половина расхождения между значениями, полученными из SLR и LLR для n ̇ из-за рассеяния справедливыми приливами на Земле (Williams et al., 2001). Другие полученные значения LLR для n ̇ были представлены Newhall et al. (1988), Dickey et al. (1994a) и Chapront et al. , (2002).

По a priori принимая значение приливного ускорения n ̇ Луны, наблюдения за лунными и солнечными затмениями могут быть использованы для определения векового увеличения продолжительности дня за последние несколько тысяч лет. Самые последние повторные редукции наблюдений за лунными и солнечными затмениями на предмет изменений LOD сделаны Стефенсоном и Моррисоном (1995) и Моррисоном и Стивенсоном (2001).Помимо наблюдений за затмениями в период с 700 г. до н.э. до 1600 г. н.э., они также использовали наблюдения за лунным затмением в период 1600–1955,5 гг. И оптические астрометрические и космически-геодезические измерения за период 1955,5–1990 гг. Приняв значение -26,0 ″ / cy 2 для n ̇, Моррисон и Стивенсон (2001) обнаружили, что LOD увеличивался со скоростью +1,80 ± 0,1 мс / цикл в среднем за последние 2700 лет (см. Рисунок 3 ). В дополнение к вековой тенденции Стефенсон и Моррисон (1995) и Моррисон и Стефенсон (2001) также обнаружили свидетельства флуктуации LOD, размах которого составляет около 8 мс и период около 1500 лет ( Рисунок 3 ).

Рис. 3. Вековые изменения LOD за последние 2500 лет, оцененные по данным лунных и солнечных затмений, затмения Луны, оптических астрометрических и космических геодезических наблюдений. Разница между наблюдаемым вековым трендом и трендом, вызванным приливным трением, обусловлена ​​эффектами изостатической регулировки ледников и другими процессами, такими как изменение массы ледяного покрова и сопутствующее нестерическое изменение уровня моря. Из Morrison LV и Stephenson FR (2001) Исторические затмения и изменчивость вращения Земли. Журнал геодинамики 32: 247–265.

По сохранению углового момента приливное ускорение Луны на –26,0 ″ / cy 2 должно сопровождаться увеличением продолжительности дня на +2,3 мс / cy (Stephenson and Morrison, 1995). Поскольку наблюдаемое увеличение продолжительности дня составляет всего +1,8 мс / с (Morrison and Stephenson, 2001), должен действовать какой-то другой механизм или комбинация механизмов, чтобы изменить продолжительность дня на -0,5 мс / с. Согласно уравнениям [43] и [58], изменения как осевой составляющей относительного углового момента, так и полярного момента инерции Земли могут вызвать изменение LOD.Вековая тенденция общей циркуляции флюидов, таких как атмосфера и океаны, и, следовательно, атмосферного и океанического углового момента, вряд ли сохранится в течение нескольких тысяч лет. Фактически, используя результаты 100-летнего прогона модели общей циркуляции атмосферы Центра Хэдли, de Viron et al. (2004) обнаружил, что смоделированный вековой тренд атмосферного углового момента (AAM) в течение 1870–1997 гг. Вызывает вековой тренд LOD только +0,08 мс / с.

Одним из наиболее важных механизмов, вызывающих вековой тренд LOD во временных масштабах в несколько тысяч лет, является изостатическое регулирование ледникового покрова (GIA).Изостатическая регулировка твердой Земли в ответ на уменьшающуюся нагрузку на нее после последней дегляциации вызывает изменение фигуры Земли и, следовательно, изменение LOD. Поскольку твердая Земля отскакивает в областях на высоких широтах, где раньше находилась ледовая нагрузка, фигура Земли становится менее сжатой, вращение Земли ускоряется, а LOD уменьшается. Модели GIA показывают, что его влияние на LOD очень чувствительно к предполагаемому значению вязкости нижней мантии (например.g., Wu and Peltier, 1984; Пельтье и Цзян, 1996; Vermeersen et al. , 1997; Митровица и Милн, 1998; Джонстон и Ламбек, 1999; Tamisea et al. , 2002; Сабадини и Вермеерсен, 2004). Но создав модель для профиля радиальной вязкости Земли, которая соответствует как временам послеледникового распада, так и аномалиям силы тяжести в свободном воздухе, связанным с мантийной конвекцией, Митровица и Форте (1997) обнаружили, что GIA должна вызывать вековую тенденцию в LOD, составляющую — 0,5 мс / цикл, значение, которое замечательно согласуется с тем, что необходимо для объяснения разницы между наблюдаемым вековым трендом продолжительности дня и трендом, вызванным приливной диссипацией.

Однако GIA — не единственный механизм, который вызовет вековой тренд продолжительности дня. Современное изменение массы ледников и ледникового покрова и сопутствующее изменение нестерического уровня моря также вызовут вековую тенденцию в LOD (например, Peltier, 1988; Trupin et al. , 1992; Mitrovica and Peltier, 1993; Trupin , 1993; Джеймс и Айвинс, 1995, 1997; Накада и Окуно, 2003; Този и др. , 2005). Но влияние этого механизма на LOD очень чувствительно к неизвестному в настоящее время массовому изменению ледников и ледяных щитов, особенно антарктического ледяного покрова.Принимая различные сценарии массового изменения Антарктиды, модели предсказывают, что только ее массовое изменение должно вызвать вековой тренд LOD в диапазоне от -0,72 до +0,31 мс / с (Джеймс и Айвинс, 1997). Другие механизмы, которые могут вызвать вековой тренд LOD, включают тектонические процессы, происходящие в неизостатических условиях (Vermeersen and Vlaar, 1993; Vermeersen et al. , 1994; Sabadini and Vermeersen, 2004), субдукция плит (Alfonsi and Spada, 1998). землетрясения (Chao, Gross, 1987) и деформация мантии, вызванная колебаниями давления на границе ядро-мантия, связанными с движением флюидного ядра (Fang et al., 1996; Дамберри и Блоксхэм, 2004; Greff-Lefftz et al. , 2004).

Колебание LOD за 1500-летний период, обнаруженное Стивенсоном и Моррисоном (1995) и Моррисоном и Стивенсоном (2001), в настоящее время имеет неизвестное происхождение. Однако, учитывая его большую амплитуду, которая слишком велика, чтобы быть вызванной атмосферными и океаническими процессами, но сравнима по размеру с амплитудой десятилетних вариаций, она, вероятно, вызвана (Dumberry and Bloxham, 2006) тем же ядром и мантией. взаимодействия ( см. Глава 8.12), такие как гравитационное взаимодействие (Rubincam, 2003), которые, как известно, вызывают десятилетние вариации LOD.

10.1 Угловое ускорение — Физика колледжа, главы 1-17

Сводка

  • Опишите равномерное круговое движение.
  • Объясните неравномерное круговое движение.
  • Вычислить угловое ускорение объекта.
  • Обратите внимание на связь между линейным и угловым ускорением.

Глава 6 Равномерное круговое движение и гравитация обсуждают только равномерное круговое движение, то есть движение по кругу с постоянной скоростью и, следовательно, с постоянной угловой скоростью.Напомним, что угловая скорость [латекс] \ boldsymbol {\ omega} [/ latex] была определена как временная скорость изменения угла [латекс] \ boldsymbol {\ theta}: [/ latex]

[латекс] \ boldsymbol {\ omega \: =} [/ latex] [латекс] \ boldsymbol {\ frac {\ Delta \ theta} {\ Delta {t}}}, [/ латекс]

где [latex] \ boldsymbol {\ theta} [/ latex] — это угол поворота, как показано на рисунке 1. Связь между угловой скоростью [латекс] \ boldsymbol {\ omega} [/ latex] и линейной скоростью [латекс] \ boldsymbol {v} [/ latex] также был определен в главе 6.1 Угол поворота и угловая скорость как

[латекс] \ boldsymbol {v = r \ omega} [/ латекс]

или

[латекс] \ boldsymbol {\ omega \: =} [/ latex] [латекс] \ boldsymbol {\ frac {v} {r}}, [/ latex]

, где [latex] \ boldsymbol {r} [/ latex] — радиус кривизны, также видно на рисунке 1. Согласно соглашению о знаках, направление против часовой стрелки считается положительным направлением, а направление по часовой стрелке — отрицательным.

Рис. 1. На этом рисунке показано равномерное круговое движение и некоторые из его определенных величин.

Угловая скорость непостоянна, когда фигуристка тянет на руках, когда ребенок запускает карусель из состояния покоя или когда жесткий диск компьютера замедляется до полной остановки при выключении. Во всех этих случаях существует угловое ускорение , при котором изменяется [latex] \ boldsymbol {\ omega} [/ latex]. Чем быстрее происходит изменение, тем больше угловое ускорение. Угловое ускорение [латекс] \ boldsymbol {\ alpha} [/ latex] определяется как скорость изменения угловой скорости. В форме уравнения угловое ускорение выражается следующим образом:

[латекс] \ boldsymbol {\ alpha \: =} [/ latex] [латекс] \ boldsymbol {\ frac {\ Delta \ omega} {\ Delta {t}}}, [/ латекс]

, где [latex] \ boldsymbol {\ Delta \ omega} [/ latex] — это изменение угловой скорости, а [latex] \ boldsymbol {\ Delta {t}} [/ latex] — изменение во времени.2}. [/ Latex] Если [latex] \ boldsymbol {\ omega} [/ latex] увеличивается, то [latex] \ boldsymbol {\ alpha} [/ latex] положительно. Если [latex] \ boldsymbol {\ omega} [/ latex] уменьшается, то [latex] \ boldsymbol {\ alpha} [/ latex] отрицательно.

Пример 1: Расчет углового ускорения и замедления велосипедного колеса

Предположим, что подросток кладет велосипед на спину и запускает вращение заднего колеса от состояния покоя до конечной угловой скорости 250 об / мин за 5,00 с. 2}.2}, [/ latex] сколько времени нужно, чтобы колесо остановилось?

Стратегия для (а)

Угловое ускорение можно найти непосредственно из его определения в [latex] \ boldsymbol {\ alpha = \ frac {\ Delta \ omega} {\ Delta {t}}} [/ latex], поскольку даны окончательная угловая скорость и время . Мы видим, что [latex] \ boldsymbol {\ Delta \ omega} [/ latex] составляет 250 об / мин, а [latex] \ boldsymbol {\ Delta {t}} [/ latex] составляет 5,00 с.

Решение для (а)

Вводя известную информацию в определение углового ускорения, получаем

[латекс] \ begin {array} {lcl} \ boldsymbol {\ alpha} & \ boldsymbol {=} & \ boldsymbol {\ frac {\ Delta \ omega} {\ Delta {t}}} \\ {} & \ boldsymbol {=} & \ boldsymbol {\ frac {250 \ textbf {rpm}} {5.2} [/ latex] для углового ускорения нам нужно преобразовать [latex] \ boldsymbol {\ Delta \ omega} [/ latex] из об / мин в рад / с:

[латекс] \ begin {array} {lcl} \ boldsymbol {\ Delta \ omega} & \ boldsymbol {=} & \ boldsymbol {250 \ frac {\ textbf {rev}} {\ textbf {min}} \ cdotp \ frac {2 \ pi \ textbf {rad}} {\ textbf {rev}} \ cdotp \ frac {1 \ textbf {min}} {60 \ textbf {sec}}} \\ {} & \ boldsymbol {=} & \ boldsymbol {26.2 \ textbf {rads.}} \ end {array} [/ latex]

Вводя это количество в выражение для [latex] \ boldsymbol {\ alpha}, [/ latex], получаем

[латекс] \ begin {array} {lcl} \ boldsymbol {\ alpha} & \ boldsymbol {=} & \ boldsymbol {\ frac {\ Delta \ omega} {\ Delta {t}}} \\ {} & \ boldsymbol {=} & \ boldsymbol {\ frac {26.2} \ end {array} [/ latex]

Стратегия для (б)

В этой части мы знаем угловое ускорение и начальную угловую скорость. Мы можем найти время остановки, используя определение углового ускорения и решение для [latex] \ boldsymbol {\ Delta {t}}, [/ latex], что дает

[латекс] \ boldsymbol {\ Delta {t} \: =} [/ latex] [латекс] \ boldsymbol {\ frac {\ Delta \ omega} {\ alpha}}. [/ Латекс]

Решение для (b)

Здесь угловая скорость уменьшается от [latex] \ boldsymbol {26.2}} \\ {} & \ boldsymbol {=} & \ boldsymbol {0.300 \ textbf {s.}} \ End {array} [/ latex]

Обсуждение

Обратите внимание, что угловое ускорение, когда девушка вращает колесо, небольшое и положительное; для получения заметной угловой скорости требуется 5 с. Когда она нажимает на тормоз, угловое ускорение велико и отрицательно. Угловая скорость быстро стремится к нулю. В обоих случаях отношения аналогичны тому, что происходит с линейным движением. Например, когда вы врезаетесь в кирпичную стену, происходит сильное замедление — изменение скорости велико за короткий промежуток времени.

Если бы велосипед в предыдущем примере был на колесах, а не перевернут, он сначала разогнался бы по земле, а затем остановился бы. Эту связь между круговым движением и линейным движением необходимо исследовать. Например, было бы полезно знать, как связаны линейное и угловое ускорение. При круговом движении линейное ускорение составляет касательное к окружности в интересующей точке, как показано на рисунке 2. Таким образом, линейное ускорение называется касательным ускорением [латекс] \ boldsymbol {a _ {\ textbf {t}}}.[/ латекс]

Рис. 2. При круговом движении линейное ускорение a возникает по мере изменения величины скорости: a касается движения. В контексте кругового движения линейное ускорение также называется тангенциальным ускорением a t .

Линейное или тангенциальное ускорение относится к изменениям величины скорости, но не ее направления. Из главы 6 «Равномерное круговое движение и гравитация» мы знаем, что центростремительное ускорение при круговом движении [latex] \ boldsymbol {a _ {\ textbf {c}}}, [/ latex] относится к изменениям направления скорости, но не ее величине. .Объект, совершающий круговое движение, испытывает центростремительное ускорение, как показано на рисунке 3. Таким образом, [latex] \ boldsymbol {a _ {\ textbf {t}}} [/ latex] и [latex] \ boldsymbol {a _ {\ textbf {c} }} [/ latex] перпендикулярны и независимы друг от друга. Касательное ускорение [латекс] \ boldsymbol {a _ {\ textbf {t}}} [/ latex] напрямую связано с угловым ускорением [latex] \ boldsymbol {\ alpha} [/ latex] и связано с увеличением или уменьшением скорость, но не ее направление.

Рис. 3. Центростремительное ускорение a c возникает при изменении направления скорости; он перпендикулярен круговому движению.Таким образом, центростремительное и тангенциальное ускорения перпендикулярны друг другу.

Теперь мы можем найти точную взаимосвязь между линейным ускорением [latex] \ boldsymbol {a _ {\ textbf {t}}} [/ latex] и угловым ускорением [latex] \ boldsymbol {\ alpha}. [/ Latex] Потому что линейное ускорение пропорциональна изменению величины скорости, она определена (как и в главе 2 «Одномерная кинематика») равной

.

[латекс] \ boldsymbol {a _ {\ textbf {t}} \: =} [/ latex] [латекс] \ boldsymbol {\ frac {\ Delta {v}} {\ Delta {t}}.} [/ латекс]

Для кругового движения обратите внимание, что [latex] \ boldsymbol {v = r \ omega}, [/ latex], так что

[латекс] \ boldsymbol {a _ {\ textbf {t}} \: =} [/ latex] [латекс] \ boldsymbol {\ frac {\ Delta (r \ omega)} {\ Delta {t}}.} [ / латекс]

Радиус [латекс] \ boldsymbol {r} [/ latex] постоянен для кругового движения, поэтому [латекс] \ boldsymbol {\ Delta (r \ omega) = r (\ Delta \ omega)}. [/ Latex] Таким образом,

[латекс] \ boldsymbol {a _ {\ textbf {t}} = r} [/ latex] [латекс] \ boldsymbol {\ frac {\ Delta \ omega} {\ Delta {t}}.} [/ Латекс]

По определению [латекс] \ boldsymbol {\ alpha = \ frac {\ Delta \ omega} {\ Delta {t}}}.[/ latex] Таким образом,

[латекс] \ boldsymbol {a _ {\ textbf {t}} = r \ alpha}, [/ latex]

или

[латекс] \ boldsymbol {\ alpha \: =} [/ latex] [латекс] \ boldsymbol {\ frac {a _ {\ textbf {t}}} {r}.} [/ Latex]

Эти уравнения означают, что линейное ускорение и угловое ускорение прямо пропорциональны. Чем больше угловое ускорение, тем больше линейное (тангенциальное) ускорение, и наоборот. Например, чем больше угловое ускорение ведущих колес автомобиля, тем больше ускорение автомобиля.Радиус тоже имеет значение. Например, чем меньше колесо, тем меньше его линейное ускорение для данного углового ускорения [латекс] \ boldsymbol {\ alpha}. [/ Latex]

Пример 2: Расчет углового ускорения колеса мотоцикла

Мощный мотоцикл может разогнаться от 0 до 30,0 м / с (около 108 км / ч) за 4,20 с. Каково угловое ускорение его колес радиусом 0,320 м? (См. Рисунок 4.)

Рисунок 4 . Линейное ускорение мотоцикла сопровождается угловым ускорением его колес.

Стратегия

Нам дана информация о линейных скоростях мотоцикла. Таким образом, мы можем найти его линейное ускорение [latex] \ boldsymbol {a _ {\ textbf {t}}}. [/ Latex] Тогда выражение [latex] \ boldsymbol {\ alpha = \ frac {a _ {\ textbf {t }}} {r}} [/ latex] можно использовать для определения углового ускорения.

Решение

Линейное ускорение

[латекс] \ begin {array} {lcl} \ boldsymbol {a _ {\ textbf {t}}} & \ boldsymbol {=} & \ boldsymbol {\ frac {\ Delta {v}} {\ Delta {t}} } \\ {} & \ boldsymbol {=} & \ boldsymbol {\ frac {30.2.} \ end {array} [/ latex]

Мы также знаем радиус колес. Ввод значений для [latex] \ boldsymbol {a _ {\ textbf {t}}} [/ latex] и [latex] \ boldsymbol {r} [/ latex] в [latex] \ boldsymbol {\ alpha = \ frac {a_ {\ textbf {t}}} {r}}, [/ latex] получаем

[латекс] \ begin {array} {lcl} \ boldsymbol {\ alpha} & \ boldsymbol {=} & \ boldsymbol {\ frac {a _ {\ textbf {t}}} {r}} \\ {} & \ boldsymbol {=} & \ boldsymbol {\ frac {7.14 \ textbf {m / s}} {20.320 \ textbf {m}}} \\ {} & \ boldsymbol {=} & \ boldsymbol {22.2.} \ end {array} [/ latex]

Обсуждение

Радианы безразмерны и присутствуют в любом соотношении между угловыми и линейными величинами.

До сих пор мы определили три вращательные величины — [латекс] \ boldsymbol {\ theta, \: \ omega}, [/ latex] и [латекс] \ boldsymbol {\ alpha}. [/ Latex] Эти величины аналогичны трансляционные величины [латекс] \ boldsymbol {x}, \: \ boldsymbol {v}, [/ latex] и [latex] \ boldsymbol {a}. [/ latex] В таблице 1 показаны вращательные величины, аналогичные трансляционные величины и отношения между ними.

Вращательный Трансляционный Отношения
[латекс] \ boldsymbol {\ theta} [/ латекс] [латекс] \ boldsymbol {x} [/ латекс] [латекс] \ boldsymbol {\ theta = \ frac {x} {r}} [/ латекс]
[латекс] \ boldsymbol {\ omega} [/ латекс] [латекс] \ boldsymbol {v} [/ латекс] [латекс] \ boldsymbol {\ omega = \ frac {v} {r}} [/ латекс]
[латекс] \ boldsymbol {\ alpha} [/ латекс] [латекс] \ boldsymbol {a} [/ латекс] [латекс] \ boldsymbol {\ alpha = \ frac {a _ {\ textbf {t}}} {r}} [/ latex]
Таблица 1. Вращательные и поступательные величины.

УСТАНОВКА ПОДКЛЮЧЕНИЙ: ЭКСПЕРИМЕНТ НА ​​ДОМУ


Сядьте, поставив ноги на землю, на вращающийся стул. Поднимите одну ногу так, чтобы она была разогнута (выпрямлена). Используя другую ногу, начните вращаться, отталкиваясь от земли. Прекратите толкать землю ногой, но позвольте стулу вращаться. От исходной точки, с которой вы начали, нарисуйте угол, угловую скорость и угловое ускорение вашей ноги как функцию времени в виде трех отдельных графиков.Оцените величину этих величин.

Проверьте свое понимание

1: Угловое ускорение — это вектор, имеющий как величину, так и направление. Как обозначить его величину и направление? Проиллюстрируйте на примере.

PHET EXPLORATIONS: LADYBUG REVOLUTION

Присоединяйтесь к божьей коровке и исследуйте вращательное движение. Вращайте карусель, чтобы изменить ее угол, или выберите постоянную угловую скорость или угловое ускорение.Изучите, как круговое движение связано с координатами x, y, скоростью и ускорением жука, используя векторы или графики.

Рисунок 5. Божья коровка Revolution
  • Равномерное круговое движение — это движение с постоянной угловой скоростью [латекс] \ boldsymbol {\ omega = \ frac {\ Delta \ theta} {\ Delta {t}}}. [/ Latex]
  • При неравномерном круговом движении скорость изменяется со временем, а скорость изменения угловой скорости (т.е. углового ускорения) равна [latex] \ boldsymbol {\ alpha = \ frac {\ Delta \ omega} {\ Delta {t} }}.[/ латекс]
  • Линейное или тангенциальное ускорение относится к изменениям величины скорости, но не ее направления, заданному как [latex] \ boldsymbol {a _ {\ textbf {t}} = \ frac {\ Delta {v}} {\ Delta {t} }}. [/ latex]
  • Для кругового движения обратите внимание, что [latex] \ boldsymbol {v = r \ omega}, [/ latex] так, чтобы

    [латекс] \ boldsymbol {a _ {\ textbf {t}} \: =} [/ latex] [латекс] \ boldsymbol {\ frac {\ Delta (r \ omega)} {\ Delta {t}}}. [ / латекс]

  • Радиус r постоянен для кругового движения, поэтому [латекс] \ boldsymbol {\ Delta (r \ omega) = r \ Delta \ omega}.[/ latex] Таким образом,

    [латекс] \ boldsymbol {a _ {\ textbf {t}} = r} [/ latex] [латекс] \ boldsymbol {\ frac {\ Delta \ omega} {\ Delta {t}}}. [/ Latex]

  • По определению [латекс] \ boldsymbol {\ Delta \ omega / \ Delta {t} = \ alpha}. [/ Latex] Таким образом,

    [латекс] \ boldsymbol {a _ {\ textbf {t}} = r \ alpha} [/ latex]

    или

    [латекс] \ boldsymbol {\ alpha =} [/ latex] [латекс] \ boldsymbol {\ frac {a _ {\ textbf {t}}} {r}}. [/ Latex]

Концептуальные вопросы

1: Между вращательными и поступательными физическими величинами существуют аналогии.Определите вращательный член, аналогичный каждому из следующих: ускорение, сила, масса, работа, поступательная кинетическая энергия, линейный импульс, импульс.

2: Объясните, почему центростремительное ускорение изменяет направление скорости при круговом движении, но не ее величину.

3: При круговом движении тангенциальное ускорение может изменять величину скорости, но не ее направление. Поясните свой ответ.

4: Предположим, что блюдо стоит на краю вращающейся пластины микроволновой печи.Испытывает ли она ненулевое тангенциальное ускорение, центростремительное ускорение или и то, и другое, когда: а) пластина начинает вращаться? (б) Пластина вращается с постоянной угловой скоростью? (c) Пластина замедляется до остановки?

Задачи и упражнения

1: На пике торнадо имеет диаметр 60,0 м и скорость ветра 500 км / ч. Какова его угловая скорость в оборотах в секунду?

2: Комплексные концепции

Ультрацентрифуга ускоряется от состояния покоя до 100 000 об / мин за 2 секунды.2} [/ latex] и кратные [latex] \ boldsymbol {g} [/ latex] этой точки на полных оборотах?

3: Комплексные концепции

У вас есть точильный камень (диск) весом 90,0 кг, радиусом 0,340 м и вращающимся со скоростью 90,0 об / мин, и вы прижимаете к нему стальной топор с радиальной силой 20,0 Н. 2}.[/ latex] Направление углового ускорения вдоль фиксированной оси обозначается знаком + или a -, так же как направление линейного ускорения в одном измерении обозначается знаком + или a -. Например, представьте, что гимнастка делает сальто вперед. Ее угловой момент будет параллелен циновке слева от нее. Величина ее углового ускорения будет пропорциональна ее угловой скорости (скорости вращения) и ее моменту инерции относительно оси вращения.

Задачи и упражнения

1:

[латекс] \ boldsymbol {\ omega = 0.2} [/ латекс]

(b) [латекс] \ boldsymbol {27 \ textbf {rev}} [/ латекс]

ньютоновских механик — Направление угловой скорости просто определение или имеет физическое значение?

Поскольку в вопросе упоминается многомерное пространство, я хотел дать ответ, который работает в любом многомерном пространстве, а не только в 3. Я начну с формальных математических определений, а затем свяжу их с физической интуицией.

Вращения в $ n $ -мерном пространстве образуют группу. В частности, они образуют группу, называемую специальной ортогональной группой, которая обозначается $ \ mathrm {SO} (n) $.$ \ mathrm {SO} (n) $ также является гладким многообразием, поэтому мы называем его группой Ли.

Каждая точка на многообразии имеет касательное пространство. Элементы этого касательного пространства называются касательными векторами. Интуитивно касательный вектор сообщает нам, в каком направлении двигаться и с какой скоростью двигаться в этом направлении. То есть, он дает нам скорость , как показано ниже:

Алгебра Ли группы Ли — это просто касательное пространство в единичном элементе группы.Для $ \ mathrm {SO} (n) $ единичным элементом является вращение, которое ничего не делает, то есть без вращения.

Следовательно, угловая скорость является элементом алгебры Ли $ \ mathrm {SO} (n) $, которая обозначается $ \ mathfrak {so} (n) $.

Примечание: В терминах матриц $ \ mathrm {SO} (n) $ можно представить как набор $ n \ times n $ ортогональных матриц с определителем 1, а $ \ mathfrak {so} (n) $ можно представить в виде набора антисимметричных матриц $ n \ times n $.Матричная экспонента дает нам экспоненциальное отображение от последнего к первому.

Так на что похож $ \ mathfrak {so} (n) $? Интуитивно мы можем указать любую угловую скорость $ \ omega $ следующим образом:

  • Поверните так быстро ($ a_1 $) на этой плоскости ($ p_1 $) через начало координат.
  • Быстро поверните ($ a_2 $) на этой плоскости ($ p_2 $) через начало координат.
  • и т. Д.

Каждая плоскость $ p_i $ также имеет ориентацию , которая сообщает нам, в какую сторону мы собираемся повернуться.

Короче говоря, мы можем рассматривать $ \ omega $ как взвешенную сумму $ a_1 p_1 + a_2 p_2 + \ dots $. Но что такое $ p_i $ математически? Чтобы указать плоскость, нам нужны только 2 единичных вектора (скажем, $ \ mathbf {u} $ и $ \ mathbf {v} $), как показано ниже:

Полученная плоскость — это произведение клина $ \ mathbf {u} $ и $ \ mathbf {v} $, которое обозначается $ \ mathbf {u} \ wedge \ mathbf {v} $. Переключение порядка $ \ mathbf {u} $ и $ \ mathbf {v} $ меняет ориентацию плоскости. В сочетании они уравновешивают: \ begin {align} \ mathbf {u} \ wedge \ mathbf {v} + \ mathbf {v} \ wedge \ mathbf {u} = 0 \ end {align}

Это соответствует тому факту, что если мы так быстро вращаемся в одном направлении и так же быстро в противоположном направлении, мы ничего не получим.3 $. Вот почему в 3D мы обычно описываем плоскости с помощью «нормальных векторов»: \ begin {align} \ mathbf {u} \ times \ mathbf {v} & \ stackrel {\ text {def}} {=} \ star (\ mathbf {u} \ wedge \ mathbf {v}) \ end {align}

и вращения с использованием «осей вращения» (см. Теорему Эйлера о вращении).

Этот трюк не работает в других измерениях. Например, в 2-х измерениях двойник бивектора является скаляром, поэтому мы обычно описываем 2D-вращения с помощью скаляров.

В 4-мерном пространстве происходит нечто еще более странное: не только двойник бивектора не является вектором, но есть бивекторы, которые являются , а не лезвиями.Следовательно, есть вращения в 4-мерном пространстве, которые нельзя описать как вращения на одной плоскости. Это называется двойным вращением. Примером может служить вращение, данное

\ begin {align} \ mathbf {u} \ wedge \ mathbf {v} + \ mathbf {w} \ wedge \ mathbf {x} \ end {align}

, где $ \ mathbf {u}, \ mathbf {v}, \ mathbf {w}, \ mathbf {x} $ взаимно ортогональны. На анимации ниже показано двойное вращение, действующее на единичный четырехмерный куб (конечно, в стереографической проекции в 3D):

6.1 Угол вращения и угловая скорость — Физика колледжа: OpenStax

Сводка

  • Определите длину дуги, угол поворота, радиус кривизны и угловую скорость.
  • Вычислить угловую скорость вращения колеса автомобиля.

В главе 2 «Кинематика» мы изучили движение по прямой и ввели такие понятия, как смещение, скорость и ускорение. В главе 3 «Двумерная кинематика» рассматривается движение в двух измерениях. Движение снаряда — это частный случай двумерной кинематики, в которой объект проецируется в воздух, находясь под действием силы тяжести, и приземляется на некотором расстоянии.В этой главе мы рассматриваем ситуации, когда объект не приземляется, а движется по кривой. Мы начинаем изучение равномерного кругового движения с определения двух угловых величин, необходимых для описания вращательного движения.

Когда объекты вращаются вокруг некоторой оси — например, когда компакт-диск (компакт-диск) на рисунке 1 вращается вокруг своего центра, — каждая точка в объекте движется по дуге окружности. Рассмотрим линию от центра компакт-диска до его края. Каждая яма, используемая для записи звука вдоль этой линии, перемещается под одним и тем же углом за одно и то же время.Угол поворота — это величина поворота, аналогичная линейному расстоянию. Мы определяем угол поворота [latex] \ boldsymbol {\ Delta \ theta} [/ latex] как отношение длины дуги к радиусу кривизны:

[латекс] \ boldsymbol {\ Delta \ theta \: =} [/ latex] [латекс] \ boldsymbol {\ frac {\ Delta {s}} {r}}. [/ Latex]

Рис. 1. Все точки на компакт-диске движутся по дугам окружности. Все углубления вдоль линии от центра к краю перемещаются на один и тот же угол Δ θ за время Δ t . Рис. 2. Радиус круга повернут на угол Δ θ . Длина дуги Δ s описана на окружности.

Длина дуги [latex] \ boldsymbol {\ Delta {s}} [/ latex] — это расстояние, пройденное по круговой траектории, как показано на рисунке 2. Обратите внимание, что [latex] \ boldsymbol {r} [/ latex] радиус кривизны круговой траектории.

Мы знаем, что за один полный оборот длина дуги равна длине окружности радиуса [латекс] \ boldsymbol {r}.[/ latex] Длина окружности [латекс] \ boldsymbol {2 \ pi {r}}. [/ latex] Таким образом, за один полный оборот угол поворота равен

[латекс] \ boldsymbol {\ Delta \ theta \: =} [/ latex] [латекс] \ boldsymbol {\ frac {2 \ pi {r}} {r}} [/ latex] [латекс] \ boldsymbol {= \: 2 \ pi}. [/ Latex]

Этот результат является основой для определения единиц, используемых для измерения углов поворота, [latex] \ boldsymbol {\ Delta \ theta} [/ latex] равными радиан, (рад), определенным таким образом, что

[латекс] \ boldsymbol {2 \ pi \ textbf {rad} = 1 \ textbf {революция}.0} [/ латекс] [латекс] \ boldsymbol {\ pi} [/ латекс] Таблица 1. Сравнение угловых единиц. Рис. 3. Точки 1 и 2 вращаются на один и тот же угол θ ) , но точка 2 перемещается по большей длине дуги s ) , потому что она находится на большем расстоянии от центра вращения ( r ) .

Если [latex] \ boldsymbol {\ Delta \ theta = 2 \ pi \ textbf {rad}}, [/ latex], то компакт-диск сделал один полный оборот, и каждая точка на компакт-диске вернулась в исходное положение.0}. [/ Latex]

Насколько быстро вращается объект? Мы определяем угловую скорость [latex] \ boldsymbol {\ omega} [/ latex] как скорость изменения угла. В символах это

.

[латекс] \ boldsymbol {\ omega \: =} [/ latex] [латекс] \ boldsymbol {\ frac {\ Delta \ theta} {\ Delta {t}}}, [/ латекс]

, где угловое вращение [латекс] \ boldsymbol {\ Delta \ theta} [/ latex] происходит за время [latex] \ boldsymbol {\ Delta {t}}. [/ Latex] Чем больше угол поворота в данном количество времени, тем больше угловая скорость.Единицы измерения угловой скорости — радианы в секунду (рад / с).

Угловая скорость [латекс] \ boldsymbol {\ omega} [/ latex] аналогична линейной скорости [латекс] \ boldsymbol {v}. [/ Latex] Чтобы получить точное соотношение между угловой и линейной скоростью, мы снова рассмотрим яму на вращающемся компакт-диске. Эта яма перемещает длину дуги [латекс] \ boldsymbol {\ Delta {s}} [/ latex] за время [латекс] \ boldsymbol {\ Delta {t}}, [/ latex] и поэтому имеет линейную скорость

[латекс] \ boldsymbol {v \: =} [/ latex] [латекс] \ boldsymbol {\ frac {\ Delta {s}} {\ Delta {t}}}.[/ латекс]

Из [latex] \ boldsymbol {\ Delta \ theta = \ frac {\ Delta {s}} {r}} [/ latex] мы видим, что [latex] \ boldsymbol {\ Delta {s} = r \ Delta \ theta }. [/ latex] Подставляя это в выражение для [latex] \ boldsymbol {v} [/ latex], получаем

[латекс] \ boldsymbol {v \: =} [/ latex] [латекс] \ boldsymbol {\ frac {r \ Delta \ theta} {\ Delta {t}}} [/ latex] [латекс] \ boldsymbol {= \: r \ omega}. [/ latex]

Мы записываем эту взаимосвязь двумя разными способами и получаем два разных вывода:

[латекс] \ boldsymbol {v = r \ omega \ textbf {или} \ omega \: =} [/ latex] [латекс] \ boldsymbol {\ frac {v} {r}}.[/ латекс]

Первое соотношение в [latex] \ boldsymbol {v = r \ omega \ textbf {or} \ omega \: = \ frac {v} {r}} [/ latex] утверждает, что линейная скорость [латекс] \ boldsymbol { v} [/ latex] пропорционален расстоянию от центра вращения, таким образом, он является наибольшим для точки на ободе (самый большой [латекс] \ boldsymbol {r} [/ latex]), как и следовало ожидать. Мы также можем назвать эту линейную скорость [латекс] \ boldsymbol {v} [/ latex] точки на ободе тангенциальной скоростью . Вторую взаимосвязь в [latex] \ boldsymbol {v = r \ omega \ textbf {или} \ omega \: = \ frac {v} {r}} [/ latex] можно проиллюстрировать на примере шины движущегося автомобиля.Обратите внимание, что скорость точки на ободе шины такая же, как скорость [latex] \ boldsymbol {v} [/ latex] автомобиля. См. Рис. 4. Таким образом, чем быстрее движется машина, тем быстрее вращается шина — большой [латекс] \ boldsymbol {v} [/ latex] означает большой [латекс] \ boldsymbol {\ omega}, [/ latex], потому что [латекс ] \ boldsymbol {v = r \ omega}. [/ latex] Точно так же шина большего радиуса, вращающаяся с той же угловой скоростью ([latex] \ boldsymbol {\ omega} [/ latex]), будет иметь большую линейную скорость ( [латекс] \ boldsymbol {v} [/ латекс]) для автомобиля.

Рис. 4. Автомобиль, движущийся вправо со скоростью v , имеет шину, вращающуюся с угловой скоростью ω . Скорость протектора шины относительно оси v , такая же, как если бы автомобиль был поднят домкратом. Таким образом, автомобиль движется вперед с линейной скоростью v = r ω , где r — радиус шины. Чем больше угловая скорость шины, тем больше скорость автомобиля.

Пример 1: Как быстро вращается автомобильная шина?

Рассчитайте угловую скорость автомобильной шины радиусом 0,300 м, когда автомобиль движется со скоростью [латекс] \ boldsymbol {15.0 \ textbf {м / с}} [/ latex] (примерно [латекс] \ boldsymbol {54 \ textbf {км / h}} [/ latex]). См. Рисунок 4.

Стратегия

Поскольку линейная скорость обода шины такая же, как и скорость автомобиля, мы имеем [latex] \ boldsymbol {v = 15.0 \ textbf {m / s}}. [/ Latex] Дан радиус шины быть [латексом] \ boldsymbol {r = 0.300 \ textbf {m}}. [/ Latex] Зная [латекс] \ boldsymbol {v} [/ latex] и [latex] \ boldsymbol {r}, [/ latex], мы можем использовать второе соотношение в [latex] \ жирный символ {v = r \ omega, \: \ omega = \ frac {v} {r}} [/ latex] для вычисления угловой скорости.

Решение

Для вычисления угловой скорости воспользуемся следующим соотношением:

[латекс] \ boldsymbol {\ omega \: =} [/ latex] [латекс] \ boldsymbol {\ frac {v} {r}}. [/ Latex]

Подстановка известных,

[латекс] \ boldsymbol {\ omega \: =} [/ latex] [латекс] \ boldsymbol {\ frac {15.0 \ textbf {m / s}} {0.300 \ textbf {m}}} [/ latex] [latex] \ boldsymbol {= \: 50.0 \ textbf {rad / s}}. [/ Latex]

Обсуждение

Когда мы отменяем единицы в приведенном выше вычислении, мы получаем 50,0 / с. Но угловая скорость должна иметь единицы рад / с. Поскольку радианы на самом деле безразмерны (радианы определяются как отношение расстояний), мы можем просто вставить их в ответ для угловой скорости. Также обратите внимание, что если бы землерой с гораздо большими шинами, скажем, радиусом 1,20 м, двигался с той же скоростью 15.0 м / с, его колеса будут вращаться медленнее. У них будет угловая скорость

[латекс] \ boldsymbol {\ omega = (15.0 \ textbf {m / s}) / (1.20 \ textbf {m}) = 12.5 \ textbf {rad / s}}. [/ Latex]

У [latex] \ boldsymbol {\ omega} [/ latex] и [latex] \ boldsymbol {v} [/ latex] есть направления (следовательно, они имеют угловую и линейную скорости соответственно). Угловая скорость имеет только два направления относительно оси вращения — либо по часовой стрелке, либо против часовой стрелки. Линейная скорость касается пути, как показано на рисунке 5.

ЭКСПЕРИМЕНТ НА ​​ДОМУ


Привяжите какой-либо предмет к концу веревки и поверните его по горизонтальному кругу над головой (взмахнув запястьем). Поддерживайте равномерную скорость при качании объекта и измеряйте угловую скорость движения. Какая примерная скорость объекта? Определите точку рядом с вашей рукой и выполните соответствующие измерения, чтобы рассчитать линейную скорость в этой точке. Определите другие круговые движения и измерьте их угловые скорости.

Рисунок 5. Когда объект движется по кругу, здесь муха на краю старомодной виниловой пластинки, его мгновенная скорость всегда касается круга. Направление угловой скорости в этом случае — по часовой стрелке.

PHET EXPLORATIONS: LADYBUG REVOLUTION

Рисунок 6. Ladybug Revolution

Присоединяйтесь к божьей коровке и исследуйте вращательное движение. Вращайте карусель, чтобы изменить ее угол, или выберите постоянную угловую скорость или угловое ускорение. Изучите, как круговое движение связано с координатами x, y, скоростью и ускорением жука, используя векторы или графики.

  • Равномерное круговое движение — это движение по окружности с постоянной скоростью. Угол поворота [латекс] \ boldsymbol {\ Delta \ theta} [/ latex] определяется как отношение длины дуги к радиусу кривизны:

    [латекс] \ boldsymbol {\ Delta \ theta \: =} [/ latex] [латекс] \ boldsymbol {\ frac {\ Delta {s}} {r}}, [/ латекс]

    где длина дуги [latex] \ boldsymbol {\ Delta {s}} [/ latex] — это расстояние, пройденное по круговой траектории, а [latex] \ boldsymbol {r} [/ latex] — это радиус кривизны круговой траектории.0}. [/ Latex]

  • Угловая скорость [латекс] \ boldsymbol {\ omega} [/ latex] — это скорость изменения угла,

    [латекс] \ boldsymbol {\ omega \: =} [/ latex] [латекс] \ boldsymbol {\ frac {\ Delta \ theta} {\ Delta {t}}}, [/ латекс]

    , где вращение [latex] \ boldsymbol {\ Delta \ theta} [/ latex] происходит за время [latex] \ boldsymbol {\ Delta {t}}. [/ Latex] Единицы угловой скорости — радианы в секунду (рад / с). Линейная скорость [латекс] \ boldsymbol {v} [/ latex] и угловая скорость [латекс] \ boldsymbol {\ omega} [/ latex] связаны соотношением

    [латекс] \ boldsymbol {v = r \ omega \ textbf {или} \ omega \: =} [/ latex] [латекс] \ boldsymbol {\ frac {v} {r}}.[/ латекс]

Концептуальные вопросы

1: Существует аналогия между вращательными и линейными физическими величинами. Какие вращательные величины аналогичны расстоянию и скорости?

Задачи и упражнения

1: Грузовики с полуприцепами имеют одометр на одной ступице колеса прицепа. Ступица утяжелена, поэтому она не вращается, но в ней есть шестерни для подсчета количества оборотов колеса — затем она вычисляет пройденное расстояние.Если колесо имеет диаметр 1,15 м и совершает 200 000 оборотов, сколько километров должен показывать одометр?

2: Микроволновые печи вращаются со скоростью около 6 об / мин. Что это в оборотах в секунду? Какова угловая скорость в радианах в секунду?

3: Автомобиль с шинами радиусом 0,260 м проезжает 80 000 км, прежде чем износится. Сколько оборотов делают шины без учета поддержки и любого изменения радиуса из-за износа?

4: а) Каков период вращения Земли в секундах? б) Какова угловая скорость Земли? (c) Учитывая, что Земля имеет радиус [латекс] \ boldsymbol {6.6 \ textbf {m}} [/ latex] на его экваторе, какова линейная скорость у поверхности Земли?

5: Бейсбольный питчер выносит руку вперед во время подачи, поворачивая предплечье вокруг локтя. Если скорость мяча в руке питчера составляет 35,0 м / с, а мяч находится на расстоянии 0,300 м от локтевого сустава, какова угловая скорость предплечья?

6: В лакроссе мяч выбрасывается из сетки на конец клюшки путем вращения клюшки и предплечья вокруг локтя.Если угловая скорость мяча относительно локтевого сустава составляет 30,0 рад / с, а мяч находится на 1,30 м от локтевого сустава, какова скорость мяча?

7: Грузовик с шинами радиусом 0,420 м движется со скоростью 32,0 м / с. Какова угловая скорость вращающихся шин в радианах в секунду? Что это в об / мин?

8: Комплексные концепции

При ударе по футбольному мячу игрок, выполняющий удар, вращает ногой вокруг тазобедренного сустава.

(a) Если скорость носка ботинка кикера равна 35.0 м / с, а тазобедренный сустав находится на расстоянии 1,05 м от кончика обуви, какова угловая скорость кончика обуви?

(b) Башмак находится в контакте с первоначально неподвижным футбольным мячом весом 0,500 кг в течение 20,0 мс. Какая средняя сила прилагается к футбольному мячу, чтобы придать ему скорость 20,0 м / с?

(c) Найдите максимальную дальность полета футбольного мяча, пренебрегая сопротивлением воздуха.

9: Создайте свою проблему

Рассмотрим аттракцион в парке развлечений, в котором участники вращаются вокруг вертикальной оси в цилиндре с вертикальными стенками.Как только угловая скорость достигает своего полного значения, пол опускается, и трение между стенами и пассажирами препятствует их скольжению. Постройте задачу, в которой вы вычисляете необходимую угловую скорость, которая гарантирует, что всадники не соскользнут со стены. Включите свободную схему тела одного всадника. Среди переменных, которые следует учитывать, — радиус цилиндра и коэффициенты трения между одеждой гонщика и стеной.

Глоссарий

длина дуги
[latex] \ boldsymbol {\ Delta {s}}, [/ latex] расстояние, пройденное объектом по круговой траектории
приямок
крошечное углубление на спиральной дорожке, отформованной в верхней части слоя поликарбоната CD
. 7 \ textbf {вращения}} [/ латекс]

5:

117 рад / с

7:

8:

(а) 33.3 рад / с

(б) 500 Н

(в) 40,8 м

Угловое положение | bartleby

Приложение

Угловая скорость

Имеет аналогичную линейную скорость. Линейная скорость определяется как скорость изменения линейного смещения. Точно так же угловое смещение можно определить как скорость изменения углового смещения во времени. Обозначается ω (Омега). Это дается следующим выражением.

ω = θ2 − θ1t2 − t1 = ΔθΔt

Здесь θ2 — конечное угловое положение,

θ1 — начальное угловое положение, а t2-t1 — общий временной интервал.

Вышеприведенное выражение дает среднюю угловую скорость. Для нахождения мгновенной угловой скорости мы можем использовать следующее выражение.

ωins = limΔt → 0ΔθΔt

Когда временной интервал приближается к нулю, мы получаем мгновенную скорость.

Примечание

Правило правой руки определяет направление угловой скорости: согните пальцы правой руки в направлении вращения, оставляя большой палец прямо вдоль оси, здесь ось z перпендикулярна плоскости вращения (здесь XY -плоскость), а большой палец определяет направление.Например, когда частица движется против часовой стрелки, согните пальцы правой руки против часовой стрелки, оставьте большой палец прямым (здесь по оси z), а большой палец задает направление угловой скорости.

Угловое ускорение n

Подобно случаю линейного ускорения, при круговом движении мы имеем угловое ускорение. Скорость изменения угловой скорости во времени называется угловым ускорением. Обозначается он α (альфа).

α = ω2 − ω1t2 − t1 = ΔωΔt

Чтобы получить мгновенную угловую скорость, время приближается к нулю.

αins = limΔt → 0ΔωΔt

Примечание:

Чтобы получить направление углового ускорения, если угловая скорость увеличивается, она совпадает с направлением угловой скорости, но если угловая скорость уменьшается, угловое ускорение против направления угловой скорости.

Связь между угловой скоростью и линейной скоростью

Из определения углового положения мы знаем, что

θ = sr⇒s = rθ

При дифференцировании по времени имеем

dsdt = rdθdt

Где dsdt дает линейная скорость, а dθdt дает угловую скорость.

∴ у нас есть v = rω, это представляет линейную скорость с точки зрения угловой скорости и наоборот.

Центростремительное ускорение с точки зрения угловой скорости определяется следующим выражением.

α = rω2

Касательное ускорение определяется дифференцированием, v = rω по времени,

Мы получаем αtan = rα

Примечание

Ускорение может быть, даже если скорость постоянна, это из-за изменения направления скорости. Следовательно, мы можем сделать вывод, что ускорение может происходить из-за изменения направления или величины, либо того и другого вместе.

Ускорение, возникающее в частице, движущейся с равномерным круговым движением, в результате действия общей внешней силы, действующей на частицу, известно как центростремительное ускорение.

alexxlab / 10.08.1975 / Разное

Добавить комментарий

Почта не будет опубликована / Обязательны для заполнения *